Biliary and Other

Lakukan tugas rumah & ujian kamu dengan baik sekarang menggunakan Quizwiz!

After receiving lactulose the day before, the client reports having seven loose stools in the past 12 hours. Based on this data, what laboratory findings would the nurse expect? A. Hypokalemia B. Hyponatremia C. Hypercalcemia D. Hyperglycemia

...ANS: A Because lactulose can cause the client to have several loose stools daily, the nurse should monitor serum electrolyte levels, particularly the serum potassium level for hypokalemia.

Which is the priority assessment in the client experiencing regurgitation? A. Auscultation for crackles B. Inspection of the oral cavity C. Palpation of the cervical lymph nodes D. Culture of the throat for bacterial infection

...ANS: A The client with regurgitation is at risk of aspiration, pneumonia, and bronchitis. The nurse should auscultate the lungs for crackles, an indication of aspiration.

A client had a transhepatic biliary catheter placed 3 days ago. Which clinical manifestation would indicate that the procedure was successful? A. The client's sclera remains icteric. B. The client's stools are brown in color. C. The client's urine is a dark amber color. D. The client's catheter has blood return on aspiration.

...ANS: B A transhepatic biliary catheter decompresses extrahepatic ducts to promote the flow of bile. When bile flows normally, it reaches the large intestine where bile is converted to urobilinogen, coloring the stools brown.

The client with esophageal reflux who experiences regurgitation while lying flat is at risk for which complication? A. Erosion B. Bleeding C. Aspiration D. Odynophagia

...ANS: C Regurgitation of stomach contents while the client is recumbent poses a risk of aspiration for the client.

While auscultating the abdomen of a client, the nurse notes increased, loud, gurgling bowel sounds. What would be the nurse's best action? A. Palpate the abdomen. B. Percuss the abdomen. C. Notify the physician. D. Document the finding as the only action.

...ANS: C The presence of increased, loud, gurgling bowel sounds is associated with hypermotility, which can be associated with diarrhea or gastroenteritis. This sound also is heard above a complete intestinal obstruction, and the physician should be notified because the client requires measures to relieve the obstruction, if present.

For the client with cirrhosis, what nursing intervention(s) would be most appropriate to control fluid accumulation in the abdominal cavity? A. Monitoring intake and output B. Providing a low-sodium diet C. Increasing PO fluid intake D. Weighing the client daily

ANS: B A low-sodium diet is one means of controlling abdominal fluid collection. Sodium intake may be restricted to 500 mg to 1 g daily. ...

Which medication should the nurse be prepared to administer to a client with the syndrome of inappropriate antidiuretic hormone (SIADH) secretion? A. Morphine B. Demeclocycline C. Dextrose 5% in water D. Tricyclic antidepressants

ANS: B Demeclocycline, a tetracycline derivative, antagonizes antidiuretic hormone and corrects the water and sodium imbalance resulting from SIADH.

Which clinical manifestation alerts the nurse to the possibility of Graves' disease as the cause of hyperthyroidism? A. Weight loss B. Exophthalmos C. Menstrual irregularities D. Increased heart rate and blood pressure

ANS: B Graves' disease causes edema in the extraocular muscles and increased retro-orbital fat that pushes the globe of the eye forward. This exophthalmos is not a feature of hyperthyroidism from any other cause.

What is the major hormone secreted by the adrenal medulla? A. Dopamine B. Epinephrine C. Norepinephrine D. Aldosterone

ANS: B The adrenal medulla secretes norepinephrine and epinephrine in proportions of 15% and 85%, respectively.

The client just diagnosed with hyperpituitarism and acromegaly is scheduled for a hypophysectomy. Which statement made by the client indicates a need for clarification regarding this treatment? A. "I will drink whenever I feel thirsty after surgery." B. "I'm glad there will be no visible incision from this surgery." C. "I hope I can go back to wearing size 8 shoes instead of size 12." D. "I will wear slip-on shoes after surgery so I don't have to bend over."

ANS: C Although removal of the tissue that is oversecreting hormones can relieve many symptoms of hyperpituitarism, skeletal changes and organ enlargement are not reversible.

The "mustache" dressing of a client postoperative from a transsphenoidal hypophysectomy is saturated with clear, yellow-tinged fluid. What is the nurse's best first action? A. Document the finding as the only action. B. Obtain a specimen for culture. C. Test the drainage for glucose. D. Notify the physician.

ANS: C Clear, yellow-tinged drainage could be serous or could contain cerebrospinal fluid (CSF). CSF contains a relatively high concentration of glucose; serous drainage does not.

The client who is taking corticosteroids daily for severe asthma now has an elevated blood glucose level. He asks the nurse if he is now considered diabetic. What is the nurse's best response? A. "Yes, the corticosteroids have destroyed the ability of the pancreas to synthesize insulin." B. "Yes, whenever blood glucose levels are abnormally high, the condition is called diabetes." C. "No, the blood glucose level is elevated because corticosteroids increase the synthesis of glucose." D. "No, the lack of insulin is temporary and will return to a normal level when the asthma is better."

ANS: C Corticosteroids cause a "pseudodiabetes" with increased blood glucose levels by stimulating liver synthesis of glucose and suppressing glucose use by cells. The ability of the pancreas to synthesize insulin is unaffected.

What safety measure should the nurse use for the adult client who has growth hormone deficiency? A. Avoid intramuscular medications. B. Place the client in protective isolation. C. Use a lift sheet to reposition the client. D. Assist the client to move slowly from a sitting to a standing position.

ANS: C In adults, growth hormone is necessary to maintain bone density and strength. Adults with growth hormone deficiency have thin, fragile bones.

Which clinical manifestation alerts the nurse to the possibility of an endocrine disorder? A. Chronic constipation B. Weight gain of 5 pounds in the past 12 months C. Increased sense of thirst and increased urine output D. Muscle cramps after heavy exercise and during hot weather

ANS: C Increased thirst and increased urine output are associated with at least two endocrine disorders, diabetes mellitus and diabetes insipidus.

Administration of hepatitis B vaccine to a healthy 18-year-old patient has been effective when a specimen of the patient's blood reveals a. HBsAg. b. anti-HBs. c. anti-HBc IgG. d. anti-HBc IgM.

B

A 24-year-old female contracts hepatitis from contaminated food. During the acute (icteric) phase of the patient's illness, the nurse would expect serologic testing to reveal a. antibody to hepatitis D (anti-HDV). b. hepatitis B surface antigen (HBsAg). c. anti-hepatitis A virus immunoglobulin G (anti-HAV IgG). d. anti-hepatitis A virus immunoglobulin M (anti-HAV IgM).

D

the client is in the preicteric phase of hepatitis. which s/s should the nurse expect the client to exhibit during this phase? 1. clay colored stools and jaundice 2. normal appetite and pruritus 3. being afebrile and left upper quadrant pain 4. c/o fatigue and diarrhea

c/o fatigue and diarrhea: flu like symptoms are the first complaints of the client in the preicteric phase of hepatitis, which is the initial phase and may begin abruptly or insidiously

which clinical manifestations of acute hepatitis A would the nurse expect a pt. to report?

c: headache, nausea, and flu like symptoms are symptoms of hepatitis A

the client is admitted with end stage liver failure and is prescribed the laxative lactulose (chronulac). which statement indicates the client needs more teaching concerning this med? 1. i should have two to three soft stools a day 2. i must check my ammonia level daily 3. if i have diarrhea, i will call my dr 4. i should check my stool for any blood

i must check my ammonia level daily: there is no instrument used at home to test daily ammonia levels. the ammonia level is a serum level requiring venipuncture and laboratory diagnostic equipment

the nurse is preparing to administer am medications to clients. which medication should the nurse question before administering? 1. pancreatic enzymes to the client who has finished breakfast 2. the pain medication, morphine, to the client who has a respiratory rate of 20 3. the loop diuretic to the client who has a serum potassium level of 3.9 mEq/L 4. the beta blocker to the client who has an apical pulse of 68

pancreatic enzymes to the client who has finished breakfast: pancreatic enzymes must be administered with meals to enhance the digestion of starches and fats in the gastrointestinal tract

the school nurse is discussing methods to prevent an outbreak of hepatitis A with a group of high school teachers. which action is the most imp to teach the high school teachers? 1. do not allow atudents to eat or drink after each other 2. drink bottled water as much as possible 3. encourage protected sexual activity 4. sing the happy birthday song while washing hands

sing the happy birthday song while washing hands: hep A is transmitted via the fecal oral route. good hand washing helps to prevent its spread. singing the happy birthday song takes approximately 30 seconds, which is how long an individual should wash his or her hands

the nurse identifies the client problem "excess fluid volume" for the client in liver failure. which short term goal would be most appropriate for this problem? 1. the client will not gain more than 2kg a day 2. the client will have no increase in abdominal girth 3. the clients v/s will remain WNL 4. the client will receive a low sodium diet

the client will have no increase in abdominal girth: excess fluid volume could be secondary to portal hypertension. therefore, no increase in abdominal girth would be an appropriate short term goal, indicating no excess of fluid volume

the client with hepatitis asks the nurse, i went to an herbalist, who recommended i take milk thistle. what do you think about the herb? which statement is the nurses best response? 1. you are concerned about taking an herb 2. the herb has been used to treat liver disease 3. I would not take anything that is not prescribed 4. why would you want to take any herbs

the herb has been used to treat liver disease: milk thistle has an active ingredient, silymarin, which has been used to treat liver disease for more than 2000 years. it is a powerful oxidant and promotes liver cell growth

the client dx with liver problems asks the nurse, why are my stools clay colored? on which scientific rationale should the nurse base the response? 1. there is an increase in serum ammonia level. 2. the liver is unable to excrete bilirubin 3. the liver is unable to metabolize fatty foods 4. a damaged liver cannot detoxify vitamins

the liver is unable to excrete bilirubin: bilirubin, the by product of red blood cell destruction, is metabolized in the liver and excreted via the feces, which causes the feces to be brown in color, if the liver is damaged, the bilirubin is excreted via the urine and skin

The nurse is preparing a client for a CT (computed tomography) scanning of the abdomen with contrast. What question should be asked before the examination? A. "Are you allergic to iodine or seafood?" B. "Have you had anything to eat or drink within the past 12 hours?" C. "Have you finished drinking all the required fluid?" D. "Can you tolerate being tilted from side to side during the procedure?"

...ANS: A Allergies to iodine or seafood can mean a cross-allergic reaction to the contrast dye used for CT scans. Clients reporting such allergies are scheduled for CT without contrast to avoid anaphylactic reactions.

In the client with Crohn's disease experiencing severe diarrhea, what should the nurse monitor for as the priority assessment? A. Cardiac dysrhythmias B. Skin irritation C. Anorexia D. Anemia

...ANS: A Although the client with severe diarrhea may experience skin irritation and anemia, the client is most at risk of cardiac dysrhythmias secondary to potassium and magnesium loss from severe diarrhea. The client should have electrolyte levels monitored, and electrolyte replacement may be necessary.

1. For a client in hepatic coma, which outcome would be the most appropriate? A. The client is oriented to time, place, and person. B. The client exhibits no ecchymotic areas. C. The client increases oral intake to 2,000 calories/day. D. The client exhibits increased serum albumin level.

1. Answer: A. The client is oriented to time, place, and person. Hepatic coma is the most advanced stage of hepatic encephalopathy. As hepatic coma resolves, improvement in the client's level of consciousness occurs. The client should be able to express orientation to time, place, and person. Ecchymotic areas are related to decreased synthesis of clotting factors. Although oral intake may be related to level of consciousness, it is more closely related to anorexia. The serum albumin level reflects hepatic synthetic ability, not level of consciousness.

Which client is most at risk for the development of gallstones? A. 22-year-old woman who is 1 month postpartum B. 65-year-old woman after a liquid protein diet C. 70-year-old man with peptic ulcer disease D. 33-year-old man with type 2 diabetes

...ANS: B Liquid protein diets increase susceptibility to gallstones by releasing cholesterol from tissues, which is then excreted as crystals in the bile.

the nurse is to administer promethazine 12.5 mg IM and has 50mg/mL on hand. how many mL should be drawn up?

0.25mL

One of the most challenging nursing interventions to promote healing in the patient with viral hepatitis is a. providing adequate nutritional intake b. promoting strict bed rest during the icteric phase c. providing pain relief without using liver metabolized drugs. d. providing quiet diversional activities during periods of fatigue

A- Adequate nutrition is especially important in promoting regeneration of liver cells, but the anorexia of viral hepatitis is often severe, requiring creative and innovative nursing interventions. Strict bed rest is not usually required, and the patient usually has only minor discomfort from with hepatitis. Diversional activities may be required to promote psychologic rest but not during periods of fatigue

34. All of these orders are received for a patient who has vomited 1500 ml of bright red blood. Which order will the nurse act on first? a. Infuse 1000 ml of lactated Ringer's solution. b. Administer IV famotidine (Pepcid) 40 mg. c. Insert NG tube and connect to suction. d. Type and cross match for 4 units of packed red blood cells.

Answer: A Rationale: Because the patient has vomited a large amount of blood, correction of hypovolemia and prevention of hypovolemic shock are the priorities. The other actions are also important to implement quickly but are not the highest priorities. Cognitive Level: Application Text Reference: p. 996 Nursing Process: Implementation NCLEX: Physiological Integrity

A 67-year-old male patient with acute pancreatitis has a nasogastric (NG) tube to suction and is NPO. Which information obtained by the nurse indicates that these therapies have been effective? a. Bowel sounds are present. b. Grey Turner sign resolves. c. Electrolyte levels are normal. d. Abdominal pain is decreased.

D

The nurse will ask a 64-year-old patient being admitted with acute pancreatitis specifically about a history of a. diabetes mellitus. b. high-protein diet. c. cigarette smoking. d. alcohol consumption.

D

Which data will the nurse monitor in relation to the 4+ pitting edema assessed in a patient with cirrhosis? a. Hemoglobin b. Temperature c. Activity level d. Albumin level

D

Which finding indicates to the nurse that a patient's transjugular intrahepatic portosystemic shunt (TIPS) placed 3 months ago has been effective? a. Increased serum albumin level b. Decreased indirect bilirubin level c. Improved alertness and orientation d. Fewer episodes of bleeding varices

D

Identify the prophylactic immunologic agents that are used for the following: a. pre-exposure protection to HBV b. post-exposure protection to HBV

a. hepatitis B vaccine (Recombivax HB) b. hepatitis B immune globulin (HBIG) and hepatitis B vaccine

the nurse reinforces teaching for a pt after a cholecystectomy on a low fat diet. the nurse will know that the pt understands the diet if which of the following menu items is selected? a. roast chick rice gelatin dessert b. cream of chick soup, milk, gelatin dessert c. meat loaf, mashed potatoes with small amount of gravy, green beans d. turkey and cheese sandwich on whole grain bread, apply, milk

a: roast chicken, rice, and gelatin dessert is a low fat meal

a client with pancreatitis has a hx of alcohol abuse. the nurse will observe the client for agitation, nausea, vomiting, delirium tremens, and visual, auditory and tactile hallucinations. these are indications of: 1. possible cirrhosis of the liver 2. alcohol withdrawal 3. depression 4. suicidal thoughts

alcohol withdrawal

what are effective mechanisms to prevent and control the spread of hep A in any setting? select all that apply 1. education 2. hand washing 3. avoidance of illicit drug use 4. immunizations for hep B virus 5. drink the water while visiting other countries 6. do not drink water while visiting other countries

education, hand washing, do not drink water while visiting other countries: education of children and parents is imp. hand washing is an effective method of prevention. refrain from drinking water while visiting other countries

which instruction should the nurse discuss with the client who is in the icteric phase of hep C? 1. decrease alcohol intake 2. encourage rest periods 3. eat a large evening meal 4. drink diet drinks and juices

encourage rest periods: adequate rest is needed for maintaining optimal immune function

a client has been given a dx of hep A. the nurse would use which precautions for preventing the transmission of the disease to self and others? 1. reverse isolation 2. mask and gown 3. universal precautions 4. isolation precautions for trash

universal precautions

a client has been given a dx of acute pancreatitis. the nurse will assess this client for: 1. hyperkalemia 2. metabolic acidosis 3. hypocalcemia 4. hyperglycemia

hyperglycemia: or hypoglycemia could occur if damage is done to the islets of langerhans

the client dx with acute pancreatitis is being discharged home. which statement by the client indicates the teaching has been effective? 1. i should decrease my intake of coffee, tea, and cola 2. i will eat a low fat diet and avoid spicy foods 3. i will check my amylase and lipase levels daily 4. i will return to work tomorrow but take it easy

i will eat a low fat diet and avoid spicy foods: high fat and spicy foods stimulate gastric and pancreatic secretions and may precipitate an acute pancreatic attack

4. The hospital administrator had undergone percutaneous transhepatic cholangiography. which assessment finding indicates complication after the operation? A. Fever and chills B. Hypertension C. Bradycardia D. Nausea and diarrhea

4. Answer: A. Fever and chills Septicemia is a common complication after a percutaneous transhepatic cholangiography. Evidence of fever and chills, possibly indicative of septicemia, is important. Hypotension, not hypertension, is associated with septicemia. Tachycardia, not bradycardia, is most likely to occur. Nausea and diarrhea may occur but are not classic signs of sepsis.

When caring for a patient with a biliary obstruction, the nurse will anticipate administering which vitamin supplements (select all that apply)? A. Vitamin A B. Vitamin D C. Vitamin E D. Vitamin K E. Vitamin B

A. Vitamin A B. Vitamin D C. Vitamin E D. Vitamin K Biliary obstruction prevents bile from entering the small intestine and thus prevents the absorption of fat-soluble vitamins. Vitamins A, D, E, and K are all fat-soluble and thus would need to be supplemented in a patient with biliary obstruction.

Identify the rationales for the following interventions in treating the cirrhotic patient with hepatic encephalopathy a. Lactulose (Cephulac) b. Neomycin c. Eliminating blood from the GI tract

A. reduction of ammonia formation by decreasing absorption of ammonia from bowel B. reduction of ammonia formation by reducing bacterial flora that produce ammonia C. Reduction of ammonia formation by removing red blood cells as a source of protein

A 53-year-old patient is being treated for bleeding esophageal varices with balloon tamponade. Which nursing action will be included in the plan of care? a. Instruct the patient to cough every hour. b. Monitor the patient for shortness of breath. c. Verify the position of the balloon every 4 hours. d. Deflate the gastric balloon if the patient reports nausea.

B: The most common complication of balloon tamponade is aspiration pneumonia

The patient with right upper quadrant abdominal pain has an abdominal ultrasound that reveals cholelithiasis. What should the nurse expect to do for this patient? A. Prevent all oral intake. B. Control abdominal pain. C. Provide enteral feedings. D. Avoid dietary cholesterol.

B. Control abdominal pain. Patients with cholelithiasis can have severe pain, so controlling pain is important until the problem can be treated. NPO status may be needed if the patient will have surgery but will not be used for all patients with cholelithiasis. Enteral feedings should not be needed, and avoiding dietary cholesterol is not used to treat cholelithiasis

The patient with sudden pain in the left upper quadrant radiating to the back and vomiting was diagnosed with acute pancreatitis. What intervention(s) should the nurse expect to include in the patient's plan of care? A. Immediately start enteral feeding to prevent malnutrition. B. Insert an NG and maintain NPO status to allow pancreas to rest. C. Initiate early prophylactic antibiotic therapy to prevent infection. D. Administer acetaminophen (Tylenol) every 4 hours for pain relief.

B. Insert an NG and maintain NPO status to allow pancreas to rest. Initial treatment with acute pancreatitis will include an NG tube if there is vomiting and being NPO to decrease pancreatic enzyme stimulation and allow the pancreas to rest and heal. Fluid will be administered to treat or prevent shock. The pain will be treated with IV morphine because of the NPO status. Enteral feedings will only be used for the patient with severe acute pancreatitis in whom oral intake is not resumed. Antibiotic therapy is only needed with acute necrotizing pancreatitis and signs of infection.

The nurse is caring for a 73-year-old man who has cirrhosis. Which data obtained by the nurse during the assessment will be of most concern? a. The patient complains of right upper-quadrant pain with palpation. b. The patient's hands flap back and forth when the arms are extended. c. The patient has ascites and a 2-kg weight gain from the previous day. d. The patient's skin has multiple spider-shaped blood vessels on the abdomen.

B: Asterixis indicates that the patient has hepatic encephalopathy, and hepatic coma may occur.

The nurse will plan to teach the patient diagnosed with acute hepatitis B about a. side effects of nucleotide analogs. b. measures for improving the appetite. c. ways to increase activity and exercise. d. administering alpha-interferon (Intron A).

B: Maintaining adequate nutritional intake is important for regeneration of hepatocytes.

A serum potassium level of 3.2 mEq/L (3.2 mmol/L) is reported for a patient with cirrhosis who has scheduled doses of spironolactone (Aldactone) and furosemide (Lasix). due. Which action should the nurse take? a. Administer both drugs. b. Administer the spironolactone. c. Withhold the spironolactone and administer the furosemide. d. Withhold both drugs until discussed with the health care provider.

B: Spironolactone is a potassium-sparing diuretic and will help increase the patient's potassium level

To detect possible complications in a patient with severe cirrhosis who has bleeding esophageal varices, it is most important for the nurse to monitor a. bilirubin levels. b. ammonia levels. c. potassium levels. d. prothrombin time.

B: The protein in the blood in the gastrointestinal (GI) tract will be absorbed and may result in an increase in the ammonia level because the liver cannot metabolize protein very well

4. A nurse is reviewing the health record of a client who has pancreatitis. The physical exam report by the provider indicates the presence of Cullen's sign. Which of the following is an appropriate action by the nurse to identify this finding? A. Tap lightly at the costovertebral margin on the client's back. B. Palpate the client's right lower quadrant. C. Inspect the skin around the umbilicus. D. Auscultate the area below the client's scapula.

C. CORRECT: Cullen's sign is indicated by a bluish-grey discoloration in the periumbilical area.

5. A nurse in a clinic is reviewing the laboratory reports of a client who has suspected cholelithiasis. Which of the following is an expected finding? A. Serum albumin 4.1 g/dL B. WBC 9,511/uL C. Direct bilirubin 2.1 mg/dL D. Serum cholesterol 171 mg/dL

C. CORRECT: This finding is outside the expected reference range and is increased in the client who has cholelithiasis.

Which response by the nurse best explains the purpose of ranitidine (Zantac) for a patient admitted with bleeding esophageal varices? a. The medication will reduce the risk for aspiration. b. The medication will inhibit development of gastric ulcers. c. The medication will prevent irritation of the enlarged veins. d. The medication will decrease nausea and improve the appetite.

C: Esophageal varices are dilated submucosal veins. The therapeutic action of H2-receptor blockers in patients with esophageal varices is to prevent irritation and bleeding from the varices caused by reflux of acid gastric contents

The nurse will teach a patient with chronic pancreatitis to take the prescribed pancrelipase (Viokase) a. at bedtime. b. in the morning. c. with each meal. d. for abdominal pain.

C: Pancreatic enzymes are used to help with digestion of nutrients and should be taken with every meal.

Fulminant viral hepatitis as a complication of viral hepatitis is highest in those individuals with a. hepatitis A b. Hepatitis C c. hepatitis B accompanied with hepatitis C d. hepatitis B accompanied with hepatitis D

D- Although fulminant hepatitis can occur with hepatitis A and hepatitis C, it is more common in hepatitis B, especially in Hep B infection accompanied by infection with Hep D virus

The patient with a history of lung cancer and hepatitis C has developed liver failure and is considering liver transplantation. After the comprehensive evaluation, the nurse knows that which factor discovered may be a contraindication for liver transplantation? A. Has completed a college education B. Has been able to stop smoking cigarettes C. Has well-controlled type 1 diabetes mellitus D. The chest x-ray showed another lung cancer lesion.

D. The chest x-ray showed another lung cancer lesion. Contraindications for liver transplant include severe extrahepatic disease, advanced hepatocellular carcinoma or other cancer, ongoing drug and/or alcohol abuse, and the inability to comprehend or comply with the rigorous post-transplant course.

What is the priority nursing diagnosis for the client newly diagnosed with hyperthyroidism? A. Decreased cardiac output related to tachycardia B. Disturbed Body Image related to weight loss C. Hyperthermia related to hypermetabolism D. Fatigue related to energy depletion

ANS: A The cardiac problems of hyperthyroidism include increased systolic blood pressure, a widened pulse pressure, tachycardia, and other dysrhythmias. The goals of nonsurgical management are to decrease the effect of thyroid hormone on cardiac function and to reduce thyroid hormone secretion.

What would be the effect on the client's hormone response to a naturally occurring hormone if the client were taking a drug that "blocked" that hormone's receptor site? A. The client's response would indicate greater hormone metabolism. B. The client's response would resemble decreased hormone activity. C. The client's response would resemble increased hormone activity. D. The client's response would be unchanged.

ANS: B Hormones cause an activity in the target tissues by binding with their specific cellular receptor sites, thereby changing the cell's activity. When the receptor sites are occupied by other substances that block hormone binding, the cell's response is the same as when there is a decreased level of the hormone.

Which client responses demonstrate to the nurse that treatment for diabetes insipidus is effective? A. Urine output is increased; specific gravity is increased. B. Urine output is increased; specific gravity is decreased. C. Urine output is decreased; specific gravity is increased. D. Urine output is decreased; specific gravity is decreased.

ANS: C Diabetes insipidus causes urine output to be greatly increased, with a low urine osmolarity, as evidenced by a low specific gravity. Effective treatment results in a decreased urine output that is more concentrated, as evidenced by an increased specific gravity.

a postop cholecystectomy client is receiving a clear liquid diet. which foods are allowed? 1. jello, cream of chick soup, milk 2. oatmeal, oj, vanilla pudding 3. water, coffee or tea, sherbert, pureed bananas 4. a grape popsicle, plain gelatin, apple juice, water

a grape popsicle, plain gelatin, apple juice, water

which client problem has priority for the client dx with acute pancreatitis? 1. risk for fluid volume deficit 2. alteration in comfort 3. imbalanced nutrition: less than body requirements 4. knowledge deficit

alteration in comfort: autodigestion of the pancreas results in severe epigastric pain, accompanied by nausea, vomiting, abdominal tenderness and muscle guarding

the client is immediate postprocedure endoscopic retrograde cholangiopancreatogram (ERCP). which intervention should the nurse implement? 1. assess for rectal bleeding, 2, increase fluid intake 3. assess gag reflex 4. keep in supine position

assess gag reflex: the gag reflex will be suppressed as a result of the local anesthesia applied to the throat to insert the endoscope into the esophagus; therefore, the gag reflex must be assessed prior to allowing the client to resume eating or drinking

a nurse is teaching a client with newly dx hep A. the client asks how did i catch this? the best response is what? 1. blood or contaminated needles 2. body fluids 3. sexual intercourse 4. feces or contaminated food or water

feces or contaminated food or water

two days after her cholecystectomy, a client has been experiencing nausea and vomiting. the client has a t tube in place. for what electrolyte imbalance will the nurse monitor? 1. hypernatremia 2. hyperkalemia 3. hypervolemia 4. hypokalemia

hypokalemia: gastric secretions are high in postassium

a nurse is doing preop teaching for a client who will have a cholecystectomy. the nurse teaches that special care must be taken to prevent what postop complication? 1. hypostatic pneumonia 2. thrombosis 3. hemorrhage 4. paralytic ileus

hypostatic pneumonia: the incision is located below the ribcage; therefore the coughing and deep breathing necessary to prevent pneumonia are especially difficult post op

a 38 y/o client has chronic cirrhosis and is jaundiced. to decrease pruritus from the jaundice, the nurse would: 1. use wool blankets 2. keep room temp above 75.8 3. keep the room temp below 70.8 4. use lots of soap in the bathwater

keep the room temp below 70.8: a decrease in sweating decreases pruritus

a 48 y/o client has a dx of dumping syndrome after gastric surgery. it would be most imp for the client to: 1. drink concentrated carbohydrates such as milkshakes between meals 2. lie down after meals 3. eat large well balanced meals 4. drink fluids with meals

lie down after meals: to decrease the effect of gravity

the client is dx with acute pancreatitis. which health care providers admitting order should the nurse question? 1. bedrest with bathroom privleges. 2. initiate iv therapy of D5W at 125 mL/hr 3. weigh client daily 4. low fat, low carb diet

low fat, low carb diet: the client will be NPO, which will decrease stimulation of the pancreatic enzymes, resulting in decreased autodigestion of the pancreas, therefore decreasing pain

the nurse writes the problem "imbalanced nutrition: less than body requirements" for the client dx with hepatitis. which intervention should the nurse include in the plan of care/ 1. provide a high calorie intake diet 2. discuss tpn 3. instruct the client to decrease salt intake 4. encourage the client to increase water intake

provide a high calorie intake diet: sufficient energy is required for healing. adequate carbohydrate intake can spare protein. the client should eat approximately 16 carbohydrate kilocalories for ea kilogram of ideal body weight daily

the nurse is developing a plan of care for a client who has a dx of acute pancreatitis. the rationale for maintaining the client on bed rest is to: 1. reduce pancreatic and gastric secretions 2. minimize the effects of hypoglycemia 3. reduce the risk of DVT 4. decrease the likelihood of orthostatic hypotension

reduce pancreatic and gastric secretions: bed rest decreases body metabolism and thus reduces pancreatic and gastric secretions in the client with acute pancreatitis

after a liver biopsy the nurse should place the client in which position? 1. right side 2. left side 3. high fowlers 4. trendelenburg

right side:when the client is placed on the right side, the biopsy site of the liver capsule is compressed against the chest wall, and the escape of blood or bile is impeded

What would be the expected clinical manifestation for a client who has excessive production of melanocyte-stimulating hormone? A. Hypoglycemia and hyperkalemia B. Irritability and insomnia C. Increased urine output D. Darkening of the skin

ANS: D Melanocyte-stimulating hormone increases the size of melanocytes in the skin and increases the amount of pigment (melanin) they produce.

the nurse is completing discharge teaching to the client dx with acute pancreatitis. which instruction should the nurse discuss with the client? 1. instruct client to decrease alcohol intake 2. explain the need to avoid all stress 3. discuss the importance of stopping smoking 4. teach the correct way to take pancreatic enzymes

discuss the importance of stopping smoking: smoking stimulates the pancreas to release pancreatic enaymes and should be stopped

the public health nurse is discussing hep B with a group in the community. which health promotion activities should the nurse discuss with the group? Select all that apply 1. do not share needles or equipment 2. use barrier protection during sex 3. get the hep B vaccine 4. obtain immune globulin injections 5. avoid any type of hepatotoxic medication

do not share needles or equipment, use barrier protection during sex, get the hep B vaccine: hep B can be transmitted by sharing any type of needles, especially those used by drug abusers. hep B can be transmitted thru sexual activity, therefore, the nurse should recommend abstinence, mutual monogamy, or barrier protection. three doses of hep B vaccine provide immunity in 90% of healthy adults

the public health nurse is teaching day care workers. which type of hepatitis is transmitted byt he fecal oral route via contaminated food, water, or direct contact with an infected person? 1. hep a 2. hep b 3. hep c 4. hep d

hep a: the hep a virus is in the stool of infected ppl and takes up to two weeks before symptoms develop

To prepare a 56-year-old male patient with ascites for paracentesis, the nurse a. places the patient on NPO status. b. assists the patient to lie flat in bed. c. asks the patient to empty the bladder. d. positions the patient on the right side.

C: The patient should empty the bladder to decrease the risk of bladder perforation during the procedure

The client has had a liver biopsy. which postprocedure intervention should the nurse implement? 1. instruct the client to void immediatly 2. keep the client NPO for 8 hours 3. place the client on the right side 4. monitor BUN and creatinine level

place the client on the right side: direct pressure is applied to the site, and then the client is placed on the right side to maintain site pressure

the client dx with end stage liver failure is admitted with hepatic encephalopathy. which dietary restriction should be implemented by the nurse to address this complication? 1. restrict sodium intake to 2g a day 2. limit oral fluids to 1500ml a day 3. decrease the daily fat intake 4. reduce protein intake to 60 to 80g a day

reduced protein intake 60-80 g a day: ammonia is a by product of protein metabolism and contributes to hepatic encephalopathy. reducing protein intake should decrease ammonia levels

the client is admitted to the medical dept. with a dx of r/o acute pancreatitis. which laboratory values should the nurse monitor to confirm this dx? 1. creatinine and BUN 2. troponin and CK-MB 3. serum amylase and lipase 4. serum bilirubin and calcium

serum amylase and lipase: serum amylase increases within 2 to 12 hrs of the onset of acute pancreatitis to 2 to 3 times normal and returns to normal in 3 to 4 days; lipase elevates and remains elevated for 7 to 14 days

a client has acute pancreatitis. to determine its severity, the nurse would look at which lab test? 1. serum transaminase 2. serum amylase 3. serum sodium 4. serum magnesium

serum amylase: severity of pancreatitis is directly correlated to the level of amylase, a pancreatic enzyme

which type of precaution should the nurse implement to protect from being exposed to any of the hepatitis viruses? 1. airborne precautions 2. standard precautions 3. droplet precautions 4. exposure precautions

standard precautions: standard precautions apply to blood, all body fluids, secretions, and excretions, except sweat, regardless of whether they contain visible blood

a nurse is teaching a client about choledocholithiasis. the nurse explains that the term means 1. infection of the bile duct 2. stones in the hepatic and bile duct 3. inflammation of the gallbladder 4. stones in the gallbladder

stones in the hepatic and bile duct

the client has end stage liver failure secondary to alcoholic cirrhosis. which complication indicates the client is at risk for developing hepatic encephalopathy? 1. gastrointestinal bleeding 2. hypoalbuminemia 3. splenomegaly 4. hyperaldosteronism

gastrointestinal bleeding: blood in the intestinal tract is digested as a protein, which increases serum ammonia levels and increases the risk of developing hepatic encephalopathy

Which finding indicates to the nurse that lactulose (Cephulac) is effective for a 72-year-old man who has advanced cirrhosis? a. The patient is alert and oriented. b. The patient denies nausea or anorexia. c. The patient's bilirubin level decreases. d. The patient has at least one stool daily.

A

Which of the following clients is most at risk of developing hepatitis B? A. 24-year-old college student who has had several sexual partners B. 54-year-old woman who takes acetaminophen daily for headaches C. 33-year-old business man who travels frequently D. 72-year-old woman who has eaten raw shellfish

...ANS: A Hepatitis B can be spread through sexual contact, needle sharing, needle sticks, blood transfusions, hemodialysis, acupuncture, and the maternal-fetal route.

The nurse administering alpha-interferon and ribavirin (Rebetol) to a patient with chronic hepatitis C will plan to monitor for a. leukopenia. b. hypokalemia. c. polycythemia. d. hypoglycemia.

A

When providing discharge teaching for the patient after a laparoscopic cholecystectomy, what information should the nurse include? A. A lower-fat diet may be better tolerated for several weeks. B. Do not return to work or normal activities for 3 weeks. C. Bile-colored drainage will probably drain from the incision. D. Keep the bandages on and the puncture site dry until it heals.

A. A lower-fat diet may be better tolerated for several weeks. Although the usual diet can be resumed, a low-fat diet is usually better tolerated for several weeks following surgery. Normal activities can be gradually resumed as the patient tolerates. Bile-colored drainage or pus, redness, swelling, severe pain, and fever may all indicate infection. The bandage may be removed the day after surgery, and the patient can shower.

4. A nurse is caring for a client who has cirrhosis. Which of the following medications can the nurse expect to administer to this client? (Select all that apply.) A. Diuretic B. Beta-blocking agent C. Opioid analgesic D. Lactulose (Cephulac) E. Sedative

A. CORRECT: Diuretics facilitate excretion of excess fluid from the body in a client who has cirrhosis. B. CORRECT: Beta-blocking agents are prescribed for a client who has cirrhosis to prevent bleeding from varices. D. CORRECT: Lactulose (Cephulac) is prescribed for a client who has cirrhosis to aid in the elimination of ammonia in the stool.

A patient with type 2 diabetes and cirrhosis asks the nurse if it would be okay to take silymarin (milk thistle) to help minimize liver damage. The nurse responds based on what knowledge? A. Milk thistle may affect liver enzymes and thus alter drug metabolism. B. Milk thistle is generally safe in recommended doses for up to 10 years. C. There is unclear scientific evidence for the use of milk thistle in treating cirrhosis. D. Milk thistle may elevate the serum glucose levels and is thus contraindicated in diabetes.

A. Milk thistle may affect liver enzymes and thus alter drug metabolism. There is good scientific evidence that there is no real benefit from using milk thistle to protect the liver cells from toxic damage in the treatment of cirrhosis. Milk thistle does affect liver enzymes and thus could alter drug metabolism. Therefore patients will need to be monitored for drug interactions. It is noted to be safe for up to 6 years, not 10 years, and it may lower, not elevate, blood glucose levels.

The client has a deficiency of all the following pituitary hormones. Which one should be addressed first? A. Growth hormone B. Luteinizing hormone C. Thyroid-stimulating hormone D. Follicle-stimulating hormone

ANS: C A deficiency of thyroid-stimulating hormone (TSH) is the most life-threatening deficiency of the hormones listed in this question. TSH is needed to ensure proper synthesis and secretion of the thyroid hormones, whose functions are essential for life.

the female nurse sticks herself with a contaminated needle. which action should the nurse implement first. 1. notify the infection control nurse 2. cleanse the area with soap and water 3. request postexposure prophylaxis 4. check the hepatitis status of the client

cleanse the area with soap and water: the nurse should first clean the needle stick with soap and water and attempt stick bleed to help remove any virus injected into the skin

36. The health care provider prescribes antacids and sucralfate (Carafate) for treatment of a patient's peptic ulcer. The nurse will teach the patient to take a. sucralfate and antacids together 30 minutes before each meal. b. antacids 30 minutes before the sucralfate. c. sucralfate at bedtime and antacids before meals. d. antacids after eating and sucralfate 30 minutes before eating.

Answer: D Rationale: Sucralfate is most effective when the pH is low and should not be given with or soon after antacid. Antacids are most effective when taken after eating. Administration of sucralfate 30 minutes before eating and antacids just after eating will ensure that both drugs can be most effective. The other regimens will decrease the effectiveness of the medications. Cognitive Level: Comprehension Text Reference: pp. 1020-1021 Nursing Process: Implementation NCLEX: Physiological Integrity

3. A nurse is caring for a client who has advanced cirrhosis with worsening hepatic encephalopahy. Which of the following is an expected assessment finding? (Select all that apply.) A. Anorexia B. Change in orientation C. Asterixis D. Ascites E. Fetor hepaticus

B. CORRECT: A change in orientation indicates worsening hepatic encephalopathy in a client who has advanced cirrhosis. C. CORRECT: Asterixis, a coarse tremor of the wrists and fingers, is observed as a late complication in a client who has cirrhosis and hepatic encephalopathy E. CORRECT: Fetor hepaticus, a fruity breath odor, is a clinical finding of worsening hepatic encephalopathy in the client who has advanced cirrhosis.

3. A nurse is completing preoperative teaching for a client who will undergo a laparoscopic cholecystectomy. Which of the following should be included in the teaching? A. "The scope will be passed through your rectum." B. "You may have shoulder pain after surgery." C. "The T-tube will remain in place for 1 to 2 weeks." D. "You should limit how often you walk for 1 to 2 weeks."

B. CORRECT: Shoulder pain occurs due to free air that is introduced into the abdomen during laparoscopic surgery.

1. A nurse is providing discharge teaching to a client who is postoperative following open cholecystectomy with T-tube placement. Which of the following instructions should the nurse include in the teaching? (Select all that apply.) A. Take baths rather than showers. B. Clamp T-tube for 1 to 2 hr before and after meals. C. Keep the drainage system above the level of the gallbladder. D. Expect to have constipation. E. Empty drainage bag every 8 hr.

B. CORRECT: The T-tube should be clamped 1 to 2 hr before and after meals to assess tolerance to food postcholecystectomy, and prior to removal. E. CORRECT: The drainage bag attached to the T-tube should be emptied every 8 hr.

2. A nurse is preparing to administer pancrelipase (Viokase) to a client who has pancreatitis. Which of the following is an appropriate nursing action? A. Administer medication 30 min after a snack. B. Offer a glass of water following medication administration. C. Administer the medication 30 min before meals. D. Sprinkle the contents on peanut butter.

B. CORRECT: The client should drink a full glass of water following administration of pancrelipase.

When taking the blood pressure (BP) on the right arm of a patient with severe acute pancreatitis, the nurse notices carpal spasms of the patient's right hand. Which action should the nurse take next? a. Ask the patient about any arm pain. b. Retake the patient's blood pressure. c. Check the calcium level in the chart. d. Notify the health care provider immediately.

C

Which finding is most important for the nurse to communicate to the health care provider about a patient who received a liver transplant 1 week ago? a. Dry palpebral and oral mucosa b. Crackles at bilateral lung bases c. Temperature 100.8° F (38.2° C) d. No bowel movement for 4 days

C

Which laboratory test result will the nurse monitor when evaluating the effects of therapy for a 62-year-old female patient who has acute pancreatitis? a. Calcium b. Bilirubin c. Amylase d. Potassium

C

The nurse is caring for a group of patients. Which patient is at highest risk for pancreatic cancer? A. A 38-year-old Hispanic female who is obese and has hyperinsulinemia B. A 23-year-old who has cystic fibrosis-related pancreatic enzyme insufficiency C. A 72-year-old African American male who has smoked cigarettes for 50 years D. A 19-year-old who has a 5-year history of uncontrolled type 1 diabetes mellitus

C. A 72-year-old African American male who has smoked cigarettes for 50 years Risk factors for pancreatic cancer include chronic pancreatitis, diabetes mellitus, age, cigarette smoking, family history of pancreatic cancer, high-fat diet, and exposure to chemicals such as benzidine. African Americans have a higher incidence of pancreatic cancer than whites. The most firmly established environmental risk factor is cigarette smoking. Smokers are two or three times more likely to develop pancreatic cancer as compared with nonsmokers. The risk is related to duration and number of cigarettes smoked.

A 38-year-old patient with cirrhosis has ascites and 4+ edema of the feet and legs. Which nursing action will be included in the plan of care? a. Restrict daily dietary protein intake. b. Reposition the patient every 4 hours. c. Place the patient on a pressure-relieving mattress. d. Perform passive range of motion daily.

C: decrease the risk for skin breakdown for this patient

The nurse recognizes early signs of hepatic encephalopathy in the patient who a. manifests asterixis b. becomes unconscious c. has increasing oliguria d. is irritable and lethargic

D- Early signs of this neurologic condition include euphoria, depression, apathy, irritability, confusion, agitation, drowsiness, and lethargy. Loss of consciousness is usually preceded by asterixis, disorientation, hyperventilation, hypothermia, and alterations in reflexes. Increasing oliguria is a sign of hepatorenal syndrome.

When teaching the patient with acute hepatitis C (HCV), the patient demonstrates understanding when the patient makes which statement? A. "I will use care when kissing my wife to prevent giving it to her." B. "I will need to take adofevir (Hepsera) to prevent chronic HCV." C. "Now that I have had HCV, I will have immunity and not get it again." D. "I will need to be checked for chronic HCV and other liver problems."

D. "I will need to be checked for chronic HCV and other liver problems." The majority of patients who acquire HCV usually develop chronic infection, which may lead to cirrhosis or liver cancer. HCV is not transmitted via saliva, but percutaneously and via high-risk sexual activity exposure. The treatment for acute viral hepatitis focuses on resting the body and adequate nutrition for liver regeneration. Adofevir (Hepsera) is taken for severe hepatitis B (HBV) with liver failure. Chronic HCV is treated with pegylated interferon with ribavirin. Immunity with HCV does not occur as it does with HAV and HBV, so the patient may be reinfected with another type of HCV.

In planning care for a patient with metastatic liver cancer, the nurse should include interventions that a. focus primarily on symptomatic and comfort measures. b. reassure the patient that chemotherapy offers a good prognosis. c. promote the patient's confidence that surgical excision of the tumor will be successful. d. provide information necessary for the patient to make decisions regarding liver transplantation.

a Rationale: Nursing intervention for a patient with liver cancer focuses on keeping the patient as comfortable as possible. The prognosis for patients with liver cancer is poor. The cancer grows rapidly, and death may occur within 4 to 7 months as a result of hepatic encephalopathy or massive blood loss from gastrointestinal (GI) bleeding.

Which dietary instructions should be included in a teaching plan for the client newly diagnosed with diverticula? A. "You should eat soft foods and smaller meals because they are better tolerated." B. "You have no dietary restrictions; you may eat anything you wish." C. "You should avoid drinking liquids with your meals." D. "You should avoid dairy products."

...ANS: A Soft foods and smaller meals assist in reducing the symptoms of pressure and reflux that accompany diverticula.

What is the nurse's best action for the client who has undergone a laparoscopic cholecystectomy and complains of "free air pain." A. Ambulate the client. B. Instruct the client to breathe deeply and cough. C. Maintain the client on bedrest with his or her legs elevated. D. Insert a rectal tube to facilitate the passage of flatus.

...ANS: A The client who has undergone a laparoscopic cholecystectomy may complain of free air pain because of the retention of carbon dioxide in the abdomen. The nurse assists the client with early ambulation to promote absorption of the carbon dioxide.

In a client admitted with cirrhosis of the liver, which serum levels would the nurse expect to be elevated? A. Serum amylase and lipase B. Serum ammonia C. Serum calcium D. Serum CEA

...ANS: B Serum ammonia levels are elevated in conditions that incur hepatocellular injury, such as cirrhosis of the liver. Increased serum amylase and lipase levels are indicators of pancreatitis. CEA levels are useful in assessing the success of cancer therapy or the recurrence of cancer.

Twenty-four hours after endoscopic retrograde cholangiopancreatography (ERCP), a client develops left upper quadrant abdominal pain and has a temperature of 101° F (38.3° C). What is the nurse's best action? A. Administer acetaminophen for control of fever and pain. B. Document the finding, because it is a normal postprocedure event. C. Notify the health care provider. D. Increase the IV fluid rate.

...ANS: C The client who has undergone an ERCP may develop complications such as perforation or sepsis manifested by fever and abdominal pain. The nurse should report these symptoms to the health care provider immediately.

What is the role of the liver in response to increased energy requirements? A. Storage of fatty acids and triglycerides B. Activation of Kupffer cells C. Storage and release of glycogen D. Removal of ammonia

...ANS: C The liver's role in carbohydrate metabolism involves the storage and release of glycogen as energy requirements change. An increase in energy requirements results in the release of glycogen.

A client with an exacerbation of ulcerative colitis has been placed on total parenteral nutrition (TPN). The client asks why nutrition is being supplied in this manner and not by mouth. What is the nurse's best response? A. "TPN contains a high percentage of glucose that is more readily absorbed into the bloodstream than into the ulcerated colon." B. "TPN will be given in addition to your meals to help you gain any weight that you may have lost through diarrhea." C. "TPN is considered an elemental formula and, as such, is easier to digest." D. "TPN will be given during this period to allow your bowel to rest."

...ANS: D Bowel rest during severe exacerbations of ulcerative colitis is part of the nonsurgical management of the disease.

What intervention should the nurse suggest to a client to prevent nighttime reflux? A. "Sleep in the right lateral decubitus position." B. "Have a light evening snack before bedtime." C. "Have alcoholic beverages early in the evening." D. "Elevate the head of the bed 8 to 12 inches for sleep."

...ANS: D Elevation of the head of the bed 8 to 12 inches for sleep is helpful in preventing nighttime reflux episodes related to the recumbent position. Wooden blocks or foam wedges can be used to achieve this level of elevation.

After a colonoscopy, a client reports that he is experiencing abdominal fullness and cramping. What is the nurse's best action? A. Insert a rectal tube to assist in passing of flatus. B. Notify the physician immediately because a bowel perforation is suspected. C. Have the client consume only liquids for 4 to 6 hours after the test. D. Explain to the client that this feeling can be expected for several hours after the test.

...ANS: D Feelings of abdominal fullness, cramping, and the passing of flatus are normally expected after a colonoscopy.

During an initial assessment, a client with gastrointestinal problems reports the use of nonsteroidal anti-inflammatory drugs (NSAIDs) three times a day for arthritis pain. What would be the nurse's best response? A. "NSAIDs are not helpful for arthritis pain." B. "NSAIDs should be taken only twice daily." C. "NSAIDs can interfere with the absorption of nutrients." D. "NSAIDs can result in ulcers or bleeding."

...ANS: D Long-term use of NSAIDs for chronic pain can precipitate peptic ulcer formation through the inhibition of prostaglandins.

18. A client diagnosed with chronic cirrhosis who has ascites and pitting peripheral edema also has hepatic encephalopathy. Which of the following nursing interventions are appropriate to prevent skin breakdown? (Select all that apply.) A. Range of motion every 4 hours B. Turn and reposition every 2 hours C. Abdominal and foot massages every 2 hours D. Alternating air pressure mattress E. Sit in chair for 30 minutes each shift

18. Answer: B, D Edematous tissue must receive meticulous care to prevent tissue breakdown. Range of motion exercises preserve joint function but do not prevent skin breakdown. Abdominal or foot massage will not prevent skin breakdown but must be cleansed carefully to prevent breaks in skin integrity. The feet should be kept at the level of heart or higher so Fowler's position should be employed. An air pressure mattress, careful repositioning can prevent skin breakdown.

10. A 52-year-old man was referred to the clinic due to increased abdominal girth. He is diagnosed with ascites by the presence of a fluid thrill and shifting dullness on percussion. After administering diuretic therapy, which nursing action would be most effective in ensuring safe care? A. Measuring serum potassium for hyperkalemia B. Assessing the client for hypervolemia C. Measuring the client's weight weekly D. Documenting precise intake and output

10. Answer: D. Documenting precise intake and output For the client with ascites receiving diuretic therapy, careful intake and output measurement is essential for safe diuretic therapy. Diuretics lead to fluid losses, which if not monitored closely and documented, could place the client at risk for serious fluid and electrolyte imbalances. Hypokalemia, not hyperkalemia, commonly occurs with diuretic therapy. Because urine output increases, a client should be assessed for hypovolemia, not hypervolemia. Weights are also an accurate indicator of fluid balance. However, for this client, weights should be obtained daily, not weekly.

11. Which assessment finding indicates that lactulose is effective in decreasing the ammonia level in the client with hepatic encephalopathy? A. Passage of two or three soft stools daily B. Evidence of watery diarrhea C. Daily deterioration in the client's handwriting D. Appearance of frothy, foul-smelling stools

11. Answer: A. Passage of two or three soft stools daily Lactulose reduces serum ammonia levels by inducing catharsis, subsequently decreasing colonic pH and inhibiting fecal flora from producing ammonia from urea. Ammonia is removed with the stool. Two or three soft stools daily indicate effectiveness of the drug. Watery diarrhea indicates overdose. Daily deterioration in the client's handwriting indicates an increase in the ammonia level and worsening of hepatic encephalopathy. Frothy, foul-smelling stools indicate steatorrhea, caused by impaired fat digestion.

12. Nurse Farrah is providing care for Kristoff who has jaundice. Which statement indicates that the nurse understands the rationale for instituting skin care measures for the client? A. "Jaundice is associated with pressure ulcer formation." B. "Jaundice impairs urea production, which produces pruritus." C. "Jaundice produces pruritus due to impaired bile acid excretion." D. "Jaundice leads to decreased tissue perfusion and subsequent breakdown."

12. Answer: C. "Jaundice produces pruritus due to impaired bile acid excretion." Jaundice is a symptom characterized by increased bilirubin concentration in the blood. Bile acid excretion is impaired, increasing the bile acids in the skin and causing pruritus. Jaundice is not associated with pressure ulcer formation. However, edema and hypoalbuminemia are. Jaundice itself does not impair urea production or lead to decreased tissue perfusion.

13. Which rationale supports explaining the placement of an esophageal tamponade tube in a client who is hemorrhaging? A. Allowing the client to help insert the tube B. Beginning teaching for home care C. Maintaining the client's level of anxiety and alertness D. Obtaining cooperation and reducing fear

13. Answer: D. Obtaining cooperation and reducing fear An esophageal tamponade tube would be inserted in critical situations. Typically, the client is fearful and highly anxious. The nurse therefore explains about the placement to help obtain the client's cooperation and reduce his fear. This type of tube is used only short term and is not indicated for home use. The tube is large and uncomfortable. The client would not be helping to insert the tube. A client's anxiety should be decreased, not maintained, and depending on the degree of hemorrhage, the client may not be alert.

14. For Rico who has chronic pancreatitis, which nursing intervention would be most helpful? A. Allowing liberalized fluid intake B. Counseling to stop alcohol consumption C. Encouraging daily exercise D. Modifying dietary protein

14. Answer: B. Counseling to stop alcohol consumption Chronic pancreatitis typically results from repeated episodes of acute pancreatitis. More than half of chronic pancreatitis cases are associated with alcoholism. Counseling to stop alcohol consumption would be the most helpful for the client. Dietary protein modification is not necessary for chronic pancreatitis. Daily exercise and liberalizing fluid intake would be helpful but not the most beneficial intervention.

15. Mr. Hasakusa is in end-stage liver failure. Which interventions should the nurse implement when addressing hepatic encephalopathy? (Select all that apply.) A. Assessing the client's neurologic status every 2 hours B. Monitoring the client's hemoglobin and hematocrit levels C. Evaluating the client's serum ammonia level D. Monitoring the client's handwriting daily E. Preparing to insert an esophageal tamponade tube F. Making sure the client's fingernails are short

15. Answer: A, C, D Hepatic encephalopathy results from an increased ammonia level due to the liver's inability to covert ammonia to urea, which leads to neurologic dysfunction and possible brain damage. The nurse should monitor the client's neurologic status, serum ammonia level, and handwriting. Monitoring the client's hemoglobin and hematocrit levels and insertion of an esophageal tamponade tube address esophageal bleeding. Keeping fingernails short address jaundice.

16. For a client with hepatic cirrhosis who has altered clotting mechanisms, which intervention would be most important? A. Allowing complete independence of mobility B. Applying pressure to injection sites C. Administering antibiotics as prescribed D. Increasing nutritional intake

16. Answer: B. Applying pressure to injection sites The client with cirrhosis who has altered clotting is at high risk for hemorrhage. Prolonged application of pressure to injection or bleeding sites is important. Complete independence may increase the client's potential for injury, because an unsupervised client may injure himself and bleed excessively. Antibiotics and good nutrition are important to promote liver regeneration. However, they are not most important for a client at high risk for hemorrhage.

17. A client with advanced cirrhosis has been diagnosed with hepatic encephalopathy. The nurse expects to assess for: A. Malaise B. Stomatitis C. Hand tremors D. Weight loss

17. Answer: C. Hand tremors Hepatic encephalopathy results from the accumulation of neurotoxins in the blood, therefore the nurse wants to assess for signs of neurological involvement. Flapping of the hands (asterixis), changes in mentation, agitation, and confusion are common. These clients typically have ascites and edema so experience weight gain. Malaise and stomatitis are not related to neurological involvement.

19. Which of the following will the nurse include in the care plan for a client hospitalized with viral hepatitis? A. Increase fluid intake to 3000 ml per day B. Adequate bed rest C. Bland diet D. Administer antibiotics as ordered

19. Answer: B. Adequate bed rest Treatment of hepatitis consists of bed rest during the acute phase to reduce metabolic demands on the liver, thus increasing blood supply and cell regeneration. Forcing fluids, antibiotics, and bland diets are not part of the treatment plan for viral hepatitis.

2. Jordin is a client with jaundice who is experiencing pruritus. Which nursing intervention would be included in the care plan for the client? A. Administering vitamin K subcutaneously B. Applying pressure when giving I.M. injections C. Decreasing the client's dietary protein intake D. Keeping the client's fingernails short and smooth

2. Answer: D. Keeping the client's fingernails short and smooth The client with pruritus experiences itching, which may lead to skin breakdown and possibly infection from scratching. Keeping his fingernails short and smooth helps prevent skin breakdown and infection from scratching. Applying pressure when giving I.M. injections and administering vitamin K subcutaneously are important if the client develops bleeding problems. Decreasing the client's dietary intake is appropriate if the client's ammonia levels are increased.

20. Spironolactone (Aldactone) is prescribed for a client with chronic cirrhosis and ascites. The nurse should monitor the client for which of the following medication-related side effects? A. Jaundice B. Hyperkalemia C. Tachycardia D. Constipation

20. Answer: B. Hyperkalemia This is a potassium-sparing diuretic so clients should be monitored closely for hyperkalemia. Diarrhea, dizziness, and headaches are other more common side effects. Tachycardia, jaundice, and constipation are not expected side effects of spironolactone (Aldactone).

3. Marie, a 51-year-old woman, is diagnosed with cholecystitis. Which diet, when selected by the client, indicates that the nurse's teaching has been successful? A. 4-6 small meals of low-carbohydrate foods daily B. High-fat, high-carbohydrate meals C. Low-fat, high-carbohydrate meals D. High-fat, low protein meals

3. Answer: C. Low-fat, high-carbohydrate meals For the client with cholecystitis, fat intake should be reduced. The calories from fat should be substituted with carbohydrates. Reducing carbohydrate intake would be contraindicated. Any diet high in fat may lead to another attack of cholecystitis.

5. When planning home care for a client with hepatitis A, which preventive measure should be emphasized to protect the client's family? A. Keeping the client in complete isolation B. Using good sanitation with dishes and shared bathrooms C. Avoiding contact with blood-soiled clothing or dressing D. Forbidding the sharing of needles or syringes

5. Answer: B. Using good sanitation with dishes and shared bathrooms Hepatitis A is transmitted through the fecal oral route or from contaminated water or food. Measures to protect the family include good handwashing, personal hygiene and sanitation, and use of standard precautions. Complete isolation is not required. Avoiding contact with blood-soiled clothing or dressings or avoiding the sharing of needles or syringes are precautions needed to prevent transmission of hepatitis B.

6. For Jayvin who is taking antacids, which instruction would be included in the teaching plan? A. "Take the antacids with 8 oz of water." B. "Avoid taking other medications within 2 hours of this one." C. "Continue taking antacids even when pain subsides." D. "Weigh yourself daily when taking this medication."

6. Answer: B. "Avoid taking other medications within 2 hours of this one." Antacids neutralize gastric acid and decrease the absorption of other medications. The client should be instructed to avoid taking other medications within 2 hours of the antacid. Water, which dilutes the antacid, should not be taken with antacid. A histamine receptor antagonist should be taken even when pain subsides. Daily weights are indicated if the client is taking a diuretic, not an antacid.

The patient is scheduled for a q12h dose of lactulose 30 grams orally. Available is an oral solution containing 5 g/10 mL. How much solution should be poured into the medication cup to give the required dose? 15 mL 30 mL 45 mL 60 mL

60mL Using the medication-calculation equation of dose desired (30 grams) divided by dose on hand (5 grams) and multipled by the quantity (10 mL), the answer is 60 mL.

7. Which clinical manifestation would the nurse expect a client diagnosed with acute cholecystitis to exhibit? A. Jaundice, dark urine, and steatorrhea B. Acute right lower quadrant (RLQ) pain, diarrhea, and dehydration C. Ecchymosis petechiae, and coffee-ground emesis D. Nausea, vomiting, and anorexia

7. Answer: D. Nausea, vomiting, and anorexia Acute cholecystitis is an acute inflammation of the gallbladder commonly manifested by the following: anorexia, nausea, and vomiting; biliary colic; tenderness and rigidity the right upper quadrant (RUQ) elicited on palpation (e.g., Murphy's sign); fever; fat intolerance; and signs and symptoms of jaundice. Ecchymosis, petechiae, and coffee-ground emesis are clinical manifestations of esophageal bleeding. The coffee-ground appearance indicates old bleeding. Jaundice, dark urine, and steatorrhea are clinical manifestations of the icteric phase of hepatitis.

8. Pierre who is diagnosed with acute pancreatitis is under the care of Nurse Bryan. Which intervention should the nurse include in the care plan for the client? A. Administration of vasopressin and insertion of a balloon tamponade B. Preparation for a paracentesis and administration of diuretics C. Maintenance of nothing-by-mouth status and insertion of nasogastric (NG) tube with low intermittent suction D. Dietary plan of a low-fat diet and increased fluid intake to 2,000 ml/day

8. Answer: C. Maintenance of nothing-by-mouth status and insertion of nasogastric (NG) tube with low intermittent suction With acute pancreatitis, the client is kept on nothing-by-mouth status to inhibit pancreatic stimulation and secretion of pancreatic enzymes. NG intubation with low intermittent suction is used to relieve nausea and vomiting, decrease painful abdominal distention, and remove hydrochloric acid. Vasopressin would be appropriate for a client diagnosed with bleeding esophageal varices. Paracentesis and diuretics would be appropriate for a client diagnosed with portal hypertension and ascites. A low-fat diet and increased fluid intake would further aggravate the pancreatitis.

9. When teaching a client about pancreatic function, the nurse understands that pancreatic lipase performs which function? A. Transports fatty acids into the brush border B. Breaks down fat into fatty acids and glycerol C. Triggers cholecystokinin to contract the gallbladder D. Breaks down protein into dipeptides and amino acids

9. Answer: B. Breaks down fat into fatty acids and glycerol Lipase hydrolyses or breaks down fat into fatty acids and glycerol. Lipase is not involved with the transport of fatty acids into the brush border. Fat itself triggers cholecystokinin release. Protein breakdown into dipeptides and amino acids is the function of trypsin, not lipase.

A 36-year-old male patient in the outpatient clinic is diagnosed with acute hepatitis C (HCV) infection. Which action by the nurse is appropriate? a. Schedule the patient for HCV genotype testing. b. Administer the HCV vaccine and immune globulin. c. Teach the patient about ribavirin (Rebetol) treatment. d. Explain that the infection will resolve over a few months.

A

The patient with cirrhosis is being taught self-care. Which statement indicates the patient needs more teaching? a. "If I notice a fast heart rate or irregular beats, it is normal for cirrhosis." b. "I need to take good care of my belly and ankle skin where it is swollen." c. "A scrotal support may be more comfortable when I have scrotal edema." d. "I can use pillows to support my head to help me breathe when I am in bed."

A If the patient with cirrhosis experiences a fast or irregular heart rate, it may be indicative of hypokalemia and should be reported to the health care provider, because this is not normal for cirrhosis. Edematous tissue is subject to breakdown and needs meticulous skin care. A scrotal support may improve comfort if there is scrotal edema. Pillows and a semi-Fowler's or Fowler's position will increase respiratory efficiency. Text Reference - p. 1024

A patient newly diagnosed with acute hep B asks about drug therapy to treat the disease. The most appropriate response by the nurse is informing the patient that a. there are no specific drugs that are effective for treating acute viral hepatitis b, only chronic hep C is treatable, primarily with antiviral agents and alpha interferon. c. no drugs can be used for treatment of viral hepatitis because of the risk of additional liver damage. d. alpha interferon combined with lamivudine (EPivir) will decrease viral load and liver damage if taken for 1 year

A- No specific drugs are effective in treating acute viral hepatitis, although supportive drugs, such as anti-emetics, sedative, or atipruritics, may be used for symptom control. Antiviral agents, such as lamivudine or ribavirin, and alpha interferon may be used for treating chronic hepatitis B or C.

The patient with cirrhosis is being taught self-care. Which statement indicates the patient needs more teaching? A. "If I notice a fast heart rate or irregular beats, this is normal for cirrhosis." B. "I need to take good care of my belly and ankle skin where it is swollen." C. "A scrotal support may be more comfortable when I have scrotal edema." D. "I can use pillows to support my head to help me breathe when I am in bed."

A. "If I notice a fast heart rate or irregular beats, this is normal for cirrhosis." If the patient with cirrhosis experiences a fast or irregular heart rate, it may be indicative of hypokalemia and should be reported to the health care provider, as this is not normal for cirrhosis. Edematous tissue is subject to breakdown and needs meticulous skin care. Pillows and a semi-Fowler's or Fowler's position will increase respiratory efficiency. A scrotal support may improve comfort if there is scrotal edema.

The nurse is caring for a woman recently diagnosed with viral hepatitis A. Which individual should the nurse refer for an immunoglobin (IG) injection? A. A caregiver who lives in the same household with the patient B. A friend who delivers meals to the patient and family each week C. A relative with a history of hepatitis A who visits the patient daily D. A child living in the home who received the hepatitis A vaccine 3 months ago

A. A caregiver who lives in the same household with the patient IG is recommended for persons who do not have anti-HAV antibodies and are exposed as a result of close contact with persons who have HAV or foodborne exposure. Persons who have received a dose of HAV vaccine more than 1 month previously or who have a history of HAV infection do not require IG.

2. A nurse is reviewing nutrition teaching for a client who has cholecystitis. Which of the following food choices can trigger cholecystitis? A. Brownie with nuts B. Bowl of mixed fruit C. Grilled turkey D. Baked potato

A. CORRECT: Foods that are high in fat, such as a brownie with nuts, can cause cholecystitis.

5. A nurse is teaching a client who has hepatitis B about home care. Which of the following should the nurse include in the teaching? (Select all that apply.) A. Limit physical activity. B. Avoid alcohol. C. Take acetaminophen for comfort. D. Wear a mask when in public places. E. Eat small frequent meals.

A. CORRECT: Limiting physical activity and taking frequent rest breaks conserves energy and assists in the recovery process for a client who has hepatitis B. B. CORRECT: Alcohol is metabolized in the liver and should be avoided by the client who has hepatitis B. E. CORRECT: A client who has hepatitis B should eat small frequent meals to promote improved nutrition due to the presence of anorexia.

The patient with cirrhosis has an increased abdominal girth from ascites. The nurse should know that this fluid gathers in the abdomen for which reasons (select all that apply)? A. There is decreased colloid oncotic pressure from the liver's inability to synthesize albumin. B. Hyperaldosteronism related to damaged hepatocytes increases sodium and fluid retention. C. Portal hypertension pushes proteins from the blood vessels, causing leaking into the peritoneal cavity. D. Osmoreceptors in the hypothalamus stimulate thirst, which causes the stimulation to take in fluids orally. E. Overactivity of the enlarged spleen results in increased removal of blood cells from the circulation, which decreases the vascular pressure.

A. There is decreased colloid oncotic pressure from the liver's inability to synthesize albumin. B. Hyperaldosteronism related to damaged hepatocytes increases sodium and fluid retention. C. Portal hypertension pushes proteins from the blood vessels, causing leaking into the peritoneal cavity. The ascites related to cirrhosis are caused by decreased colloid oncotic pressure from the lack of albumin from liver inability to synthesize it and the portal hypertension that shifts the protein from the blood vessels to the peritoneal cavity, and hyperaldosteronism which increases sodium and fluid retention. The intake of fluids orally and the removal of blood cells by the spleen do not directly contribute to ascites

When caring for a patient with liver disease, the nurse recognizes the need to prevent bleeding resulting from altered clotting factors and rupture of varices. Which nursing interventions would be appropriate to achieve this outcome (select all that apply)? A. Use smallest gauge needle possible when giving injections or drawing blood. B. Teach patient to avoid straining at stool, vigorous blowing of nose, and coughing. C. Advise patient to use soft-bristle toothbrush and avoid ingestion of irritating food. D. Apply gentle pressure for the shortest possible time period after performing venipuncture. E. Instruct patient to avoid aspirin and NSAIDs to prevent hemorrhage when varices are present.

A. Use smallest gauge needle possible when giving injections or drawing blood. B. Teach patient to avoid straining at stool, vigorous blowing of nose, and coughing. C. Advise patient to use soft-bristle toothbrush and avoid ingestion of irritating food. E. Instruct patient to avoid aspirin and NSAIDs to prevent hemorrhage when varices are present. Using the smallest gauge needle for injections will minimize the risk of bleeding into the tissues. Avoiding straining, nose blowing, and coughing will reduce the risk of hemorrhage at these sites. The use of a soft-bristle toothbrush and avoidance of irritating food will reduce injury to highly vascular mucous membranes. The nurse should apply gentle but prolonged pressure to venipuncture sites to minimize the risk of bleeding. Aspirin and NSAIDs should not be used in patients with liver disease because they interfere with platelet aggregation, thus increasing the risk for bleeding

26. The health care provider orders IV vasopressin (Pitressin) to be administered to a patient with esophageal bleeding. During administration of the drug, the nurse will monitor the patient for a. polyuria. b. metabolic alkalosis. c. intention tremors. d. chest pain.

Answer: D Rationale: Vasopressin decreases coronary artery perfusion and may cause coronary ischemia. The other symptoms are not adverse effects associated with vasopressin. Cognitive Level: Application Text Reference: p. 997 Nursing Process: Evaluation NCLEX: Physiological Integrity

The client has been taking an oral cortisol preparation for 2 years to manage an autoimmune disease. What effects does the nurse expect this therapy to have on this client's circulating levels of ACTH and aldosterone? A. Increased ACTH, increased aldosterone B. Increased ACTH, decreased aldosterone C. Decreased ACTH, increased aldosterone D. Decreased ACTH, decreased aldosterone

ANS: D Taking exogenous cortisol increases the blood levels of cortisol, causing the negative feedback loops to be inhibited. The elevated cortisol levels will suppress hypothalamic secretion of corticotropin-releasing hormone (CRH). Low levels of CRH suppress the anterior pituitary production of adrenocorticotropic hormone (ACTH). Elevated blood levels of cortisol cause increased sodium retention and water reabsorption, inhibiting aldosterone synthesis.

2. A patient with deep partial-thickness (second-degree) burns over 70% of the body experiences severe pain associated with nausea and occasional vomiting during dressing changes. To promote relief of the patient's nausea and vomiting, the nurse should a. administer the prescribed morphine sulfate before dressing changes. b. avoid performing dressing changes close to the patient's mealtimes. c. keep the patient NPO for 2 hours before and after dressing changes. d. give the ordered prochlorperazine (Compazine) before dressing changes.

Answer: A Rationale: Because the patient's nausea and vomiting are associated with severe pain, it is likely that they are precipitated by stress and pain. The best treatment will be to provide adequate pain medication before dressing changes. The nurse should avoid doing painful procedures close to mealtimes, but nausea/vomiting that occur at other times should also be addressed. Keeping the patient NPO does not address the reason for the nausea and vomiting and will have an adverse effect on the patient's nutrition. Administration of antiemetics is not the best choice for a patient with nausea caused by pain. Cognitive Level: Application Text Reference: p. 991 Nursing Process: Implementation NCLEX: Physiological Integrity

14. Which of these assessment findings in a patient with a hiatal hernia who returned from a laparoscopic Nissen fundoplication 4 hours ago is most important for the nurse to address immediately? a. The patient has absent breath sounds throughout the left lung. b. The patient complains of 6/10 (of a 0-10 scale) abdominal pain. c. The patient has decreased bowel sounds in all four quadrants. d. The patient is experiencing intermittent waves of nausea.

Answer: A Rationale: Decreased breath sounds on one side may indicate a pneumothorax, which requires rapid diagnosis and treatment. The abdominal pain and nausea should also be addressed but are not as high priority as the patient's respiratory status. The patient's decreased bowel sounds are expected after surgery and require ongoing monitoring but no other action. Cognitive Level: Application Text Reference: p. 1008 Nursing Process: Assessment NCLEX: Physiological Integrity

18. A patient has just arrived on the postoperative unit after having a laparoscopic esophagectomy for treatment of esophageal cancer. Which nursing actions should be included in the postoperative plan of care? a. Elevate the head of the bed to at least 30 degrees. b. Reposition NG tube if drainage stops or decreases. c. Notify doctor immediately about bloody NG drainage. d. Start oral fluids when patient has active bowel sounds.

Answer: A Rationale: Elevation of the head of the bed decreases the risk for reflux and aspiration of gastric secretions. The NG tube should not be repositioned without consulting with the health care provider. Bloody NG drainage is expected for the first 8 to 12 hours. A swallowing study is needed before oral fluids are started. Cognitive Level: Application Text Reference: p. 1011 Nursing Process: Planning NCLEX: Physiological Integrity

24. A patient is hospitalized with vomiting of "coffee-ground" emesis. The nurse will anticipate preparing the patient for a. endoscopy. b. angiography. c. gastric analysis testing. d. barium contrast studies.

Answer: A Rationale: Endoscopy is the primary tool for visualization and diagnosis of upper gastrointestinal (GI) bleeding. Angiography is used only when endoscopy can not be done, because it is more invasive and has more possible complications. Gastric analysis testing may help with determining the cause of gastric irritation, but it is not used for acute GI bleeding. Barium studies are helpful in determining the presence of gastric lesions, but not whether the lesions are actively bleeding. Cognitive Level: Application Text Reference: p. 997 Nursing Process: Planning NCLEX: Physiological Integrity

31. Twelve hours after undergoing a gastroduodenostomy (Billroth I) for treatment of a perforated ulcer, a patient complains of increasing abdominal pain. The nursing assessment reveals an absence of bowel sounds and 200 ml of bright red NG drainage in the last hour. The most appropriate action by the nurse at this time is to a. notify the health care provider. b. irrigate the NG tube. c. administer the ordered morphine sulfate. d. continue to monitor the NG drainage.

Answer: A Rationale: Increased pain and 200 ml of bright red NG drainage 12 hours after surgery indicate possible postoperative hemorrhage, and immediate actions such as blood transfusion and/or return to surgery are needed. Because the NG is draining, there is no indication that irrigation is needed. The patient may need morphine, but this is not the highest priority action. Continuing to monitor the NG drainage is not an adequate response. Cognitive Level: Application Text Reference: p. 1027 Nursing Process: Implementation NCLEX: Physiological Integrity

39. A patient who has intermittent epigastric distress, weight loss, and ascites is diagnosed with stomach cancer. The nurse plans care for the patient with the knowledge that these findings indicate that a. the patient has a poor prognosis with any therapy. b. surgical intervention is not indicated for the patient. c. radiation therapy is the treatment of choice for the patient. d. the patient will need a referral to hospice services.

Answer: A Rationale: Survival rate for patients with stomach cancer is low and the presence of ascites indicates metastasis and is a poor prognostic sign. The patient may be a candidate for surgery, which is the only curative treatment for stomach cancer. Radiation may be used, but it is not the treatment of choice because stomach cancers do not respond well to radiation. The patient may need a referral to hospice services, but this will depend on factors such as the patient's desires and how long the patient is projected to live. Cognitive Level: Application Text Reference: p. 1028 Nursing Process: Planning NCLEX: Physiological Integrity

A 54-year-old patient admitted with diabetes mellitus, malnutrition, osteomyelitis, and alcohol abuse has a serum amylase level of 280 U/L and a serum lipase level of 310 U/L. To what diagnosis does the nurse attribute these findings? A. Malnutrition B. Osteomyelitis C. Alcohol abuse D. Diabetes mellitus

C. Alcohol Use The patient with alcohol abuse could develop pancreatitis as a complication, which would increase the serum amylase (normal 30-122 U/L) and serum lipase (normal 31-186 U/L) levels as shown.

29. A patient with a bleeding duodenal ulcer has an NG tube in place, and the health care provider orders 30 ml of aluminum hydroxide/magnesium hydroxide (Maalox) to be instilled through the tube every hour. To evaluate the effectiveness of this treatment, the nurse a. periodically aspirates and tests gastric pH. b. measures the amount of residual stomach contents hourly. c. monitors arterial blood gas values on a daily basis. d. checks each stool for the presence of occult blood.

Answer: A Rationale: The purpose for antacids is to increase gastric pH; checking gastric pH is the most direct way of evaluating the effectiveness of the medication. Arterial blood gases may change slightly, but this does not directly reflect the effect of antacids on gastric pH. Because the patient has upper GI bleeding, occult blood in the stools will appear even after the acute bleeding has stopped. The amount of residual stomach contents is not a reflection of resolution of bleeding or of gastric pH. Cognitive Level: Application Text Reference: p. 1020 Nursing Process: Evaluation NCLEX: Physiological Integrity

8. The nurse is admitting a patient who has been diagnosed with squamous cell carcinoma of the buccal mucosa. When interviewing the patient for the health history, the nurse will ask about a. any use of tobacco by the patient. b. any history of streptococcal throat infection. c. chronic overexposure to the sun. d. recurrent herpes simplex (HSV) infections.

Answer: A Rationale: Tobacco use greatly increases the risk for oral cancer. History of acute infections such as strep throat is not a risk factor for oral cancer, although chronic irritation of the oral mucosa does increase risk. Sun exposure does not increase the risk for cancers of the buccal mucosa. Human papillomavirus infection (HPV) infection may be associated with increased risk, but HSV infection is not a risk factor for oral cancer. Cognitive Level: Comprehension Text Reference: pp. 1001-1002 Nursing Process: Assessment NCLEX: Physiological Integrity

20. A patient who is nauseated and vomiting up blood streaked fluid is admitted to the hospital with acute gastritis. When obtaining the admission health history, it will be most important for the nurse to ask the patient about a. frequency of nonsteroidal antiinflammatory drug (NSAID) use. b. family history of gastric problems. c. recent weight gain or loss. d. the amount of fat in the diet.

Answer: A Rationale: Use of an NSAID is associated with damage to the gastric mucosa, which can result in acute gastritis. Family history, recent weight gain or loss, and fatty foods are not risk factors for acute gastritis. Cognitive Level: Application Text Reference: p. 1013 Nursing Process: Assessment NCLEX: Physiological Integrity

1. Which information about a patient who has just been admitted to the hospital with nausea and vomiting will require the most rapid intervention by the nurse? a. The patient has been vomiting several times a day for the last 4 days. b. The patient is lethargic and difficult to arouse. c. The patient's chart indicates a recent resection of the small intestine. d. The patient has taken only sips of water.

Answer: B Rationale: A lethargic patient is at risk for aspiration, and the nurse will need to position the patient to decrease aspiration risk. The other information is also important to collect, but it does not require as quick action as the risk for aspiration. Cognitive Level: Application Text Reference: p. 991 Nursing Process: Assessment NCLEX: Physiological Integrity

7. When the nurse is assessing the mouth of a patient who uses smokeless tobacco for signs of oral cancer, which finding will be of most concern? a. A 3-mm ulcer on the floor of the mouth b. A red, velvety patch on the buccal mucosa c. White, curdlike plaques on the back of the tongue d. Painful blisters at the border of the lips

Answer: B Rationale: A red, velvety patch suggests erythroplasia, which has a high incidence (greater than 50%) of progression to squamous cell carcinoma. The other lesions are suggestive of acute processes (aphthous stomatitis, oral candidiasis, and herpes simplex). Cognitive Level: Comprehension Text Reference: p. 1001 Nursing Process: Assessment NCLEX: Physiological Integrity

11. When admitting a patient with a stroke who is unconscious and unresponsive to stimuli, the nurse learns from the patient's family that the patient has a history of GERD. The nurse will plan to do frequent assessment of the patient's a. bowel sounds. b. breath sounds. c. apical pulse. d. abdominal girth.

Answer: B Rationale: Because GERD may cause aspiration, the unconscious patient is at risk for developing aspiration pneumonia. Bowel sounds, abdominal girth, and apical pulse will not be affected by the patient's stroke or GERD and do not require more frequent monitoring than the routine. Cognitive Level: Application Text Reference: p. 1005 Nursing Process: Assessment NCLEX: Physiological Integrity

5. All the following orders are received for a patient who has been admitted with dehydration after 3 days of nausea and vomiting. Which order will the nurse act on first? a. Provide oral care with moistened swabs. b. Infuse normal saline at 250 ml/hr. c. Insert a 16-gauge nasogastric (NG) tube. d. Administer IV ondansetron (Zofran).

Answer: B Rationale: Because the patient has severe dehydration, rehydration with IV fluids is the priority. The other orders should be accomplished as quickly as possible after the IV fluids are initiated. Cognitive Level: Application Text Reference: pp. 993-994 Nursing Process: Implementation NCLEX: Physiological Integrity

6. A patient who is receiving chemotherapy develops a Candida albicans oral infection. The nurse will anticipate the need for a. hydrogen peroxide rinses. b. administration of nystatin (Mycostatin) oral tablets. c. the use of antiviral agents. d. referral to a dentist for professional tooth cleaning.

Answer: B Rationale: Candida albicans is treated with an antifungal such as nystatin. Oral saltwater rinses may be used but will not cure the infection. Antiviral agents are used for viral infections such as herpes simplex. Referral to a dentist is indicated for gingivitis but not for Candida infection. Cognitive Level: Application Text Reference: p. 1000 Nursing Process: Planning NCLEX: Physiological Integrity

43. The nurse suspects the possibility of Escherichia coli O157:H7 food poisoning when several individuals who have eaten in the same restaurant develop a. fever and chills. b. hemorrhagic diarrhea. c. muscular incoordination. d. nausea and vomiting.

Answer: B Rationale: E. coli O157:H7 causes hemorrhagic colitis with bloody diarrhea. Fever and chills are not typical clinical manifestations of food poisoning. Muscular incoordination is seen with botulism. Nausea and vomiting are common with some forms of food poisoning, but not with E. coli O157:H7. Cognitive Level: Comprehension Text Reference: p. 1031 Nursing Process: Assessment NCLEX: Physiological Integrity

17. Which information will the nurse include when teaching a patient with newly diagnosed GERD? a. "Peppermint tea may be helpful in reducing your symptoms." b. "You will need to keep the head of your bed elevated on blocks." c. "You should avoid eating between meals to reduce acid secretion." d. "Vigorous physical activities may increase the incidence of reflux."

Answer: B Rationale: Elevating the head of the bed will reduce the incidence of reflux while the patient is sleeping. Peppermint will lower LES pressure and increase the chance for reflux. Small, frequent meals are recommended to avoid abdominal distension. There is no need to make changes in physical activities because of GERD. Cognitive Level: Application Text Reference: p. 1007 Nursing Process: Implementation NCLEX: Physiological Integrity

23. The health care provider orders insertion of a 20-gauge orogastric tube for a patient experiencing massive hematemesis. As the nurse inserts the tube, resistance is met in advancing the tube. The appropriate action by the nurse is to a. ask the patient to hyperextend the neck. b. stop and notify the health care provider of the resistance. c. inject additional lubricant through the tube. d. withdraw the tube a few inches and then reinsert.

Answer: B Rationale: No tube should be advanced against resistance because of the risk for mucosal damage or perforation of the esophagus. Hyperextension of the neck will increase the likelihood of insertion into the trachea. Because the tube may be in the trachea, injection of lubricant may cause aspiration. Withdrawal and reinsertion of the tube will increase the risk for mucosal damage or perforation. Cognitive Level: Application Text Reference: p. 996 Nursing Process: Implementation NCLEX: Physiological Integrity

27. The health care provider orders IV ranitidine (Zantac) for a patient with an acute exacerbation of chronic peptic ulcer disease. When teaching the patient about the effect of the medication, which information will the nurse include? a. "Ranitidine constricts the blood vessels in the stomach and decreases bleeding." b. "Ranitidine decreases secretion of gastric acid." c. "Ranitidine neutralizes the acid in the stomach." d. "Ranitidine covers the ulcer with a protective material which promotes healing."

Answer: B Rationale: Ranitidine is a histamine-2 (H2) receptor blocker, which decreases the secretion of gastric acid. The response beginning, "Ranitidine constricts the blood vessels" describes the effect of vasopressin. The response beginning "Ranitidine neutralizes the acid" describes the effect of antacids. And the response beginning "Ranitidine covers the ulcer" describes the action of sucralfate (Carafate). Cognitive Level: Application Text Reference: pp. 998, 1019 Nursing Process: Implementation NCLEX: Physiological Integrity

22. A patient with chronic gastritis associated with the presence of Helicobacter pylori is treated with triple-drug therapy. The nurse explains to the patient that the drugs commonly included in this regimen include a. famotidine (Pepcid), magnesium hydroxide (Mylanta), and pantoprazole (Protonix). b. amoxicillin (Amoxil ), clarithromycin (Biaxin), and omeprazole (Prilosec). c. sucralfate (Carafate), nystatin (Mycostatin), and bismuth subsalicylate (Pepto-Bismol). d. metoclopramide (Reglan), bethanechol (Urecholine), and promethazine (Phenergan).

Answer: B Rationale: The drugs used in triple-drug therapy include a proton pump inhibitor such as omeprazole and the antibiotics amoxicillin and clarithromycin. The other combinations listed are not included in the protocol for H. pylori infection. Cognitive Level: Comprehension Text Reference: p. 1014 Nursing Process: Implementation NCLEX: Physiological Integrity

33. A patient recovering from a gastrojejunostomy (Billroth II) for treatment of a duodenal ulcer develops dizziness, weakness, and palpitations, with an urge to defecate about 20 minutes after eating. To avoid recurrence of these symptoms, the nurse teaches the patient to a. increase the amount of fluid intake with meals. b. lie down for about 30 minutes after eating. c. drink sugared fluids or eat candy after each meal. d. choose foods that are high in carbohydrates.

Answer: B Rationale: The patient is experiencing symptoms of dumping syndrome, which may be reduced by lying down after eating. Increasing fluid intake and choosing high carbohydrate foods will increase the risk for dumping syndrome. Having a sweet drink or hard candy will correct the hypoglycemia that is associated with dumping syndrome but will not prevent dumping syndrome. Cognitive Level: Application Text Reference: p. 1027 Nursing Process: Implementation NCLEX: Physiological Integrity

9. A patient with oral squamous cell carcinoma is transferred to the postoperative surgical unit after a hemiglossectomy and radical neck procedure. When planning care the nurse will anticipate the need to a. insert a long-term central venous catheter for parenteral nutrition. b. use an alphabet board to assist the patient with communication. c. administer chemotherapy starting the first postoperative day. d. reinforce pressure dressings at the surgical incision.

Answer: B Rationale: The patient will have a tracheostomy after having a radical neck procedure, and the nurse should plan ways to allow the patient to communicate. IV fluids (but not parenteral nutrition) are given for 24 to 48 hours, followed by enteral feedings. Chemotherapy is not started until after surgical wounds have healed. Pressure dressings are not used because they could obstruct the patient's airway. Cognitive Level: Application Text Reference: p. 1003 Nursing Process: Planning NCLEX: Physiological Integrity

38. A patient with acute GI bleeding is receiving normal saline IV at a rate of 500 ml/hr. Which assessment data obtained by the nurse are most important to communicate immediately to the health care provider? a. The NG suction is returning coffee-ground material. b. The patient's lungs have crackles audible to the midline. c. The patient's BP has increased to 142/94 mm Hg. d. The bowel sounds are very hyperactive in all four quadrants.

Answer: B Rationale: The patient's lung sounds indicate that pulmonary edema may be developing as a result of the rapid infusion of IV fluid and that the fluid infusion rate should be slowed. The return of coffee-ground material in an NG tube is expected for a patient with upper GI bleeding. The BP is slightly elevated but would not be an indication to contact the health care provider immediately. Hyperactive bowel sounds are common when a patient has GI bleeding. Cognitive Level: Application Text Reference: p. 999 Nursing Process: Assessment NCLEX: Physiological Integrity

30. A patient with a peptic ulcer who has an NG tube develops sudden, severe upper abdominal pain, diaphoresis, and a very firm abdomen. Which action should the nurse take next? a. Irrigate the NG tube. b. Obtain the vital signs. c. Give the ordered antacid. d. Listen for bowel sounds.

Answer: B Rationale: The patient's symptoms suggest acute perforation, and the nurse should assess for signs of hypovolemic shock. Irrigation of the NG tube, administration of antacids, or both would be contraindicated because any material in the stomach will increase the spillage into the peritoneal cavity. The nurse should assess the bowel sounds, but this is not the first action that the nurse should take. Cognitive Level: Application Text Reference: pp. 1023-1024 Nursing Process: Implementation NCLEX: Physiological Integrity

16. A 62-year-old patient who has been diagnosed with esophageal cancer tells the nurse, "I know that my chances are not very good, but I do not feel ready to die yet." Which response by the nurse is most appropriate? a. "You may have quite a few years to live still left." b. "Having this new diagnosis must be very hard for you." c. "Thinking about dying will only make you feel worse." d. "It is important that you be realistic about your prognosis."

Answer: B Rationale: This response is open-ended and will encourage the patient to further discuss feelings of anxiety or sadness about the diagnosis. Patients with esophageal cancer have only a 20% 5-year survival rate, so the response "You may have quite a few years to live still yet" is misleading. The response beginning, "Thinking about dying" indicates that the nurse is not open to discussing the patient's fears of dying. And the response beginning, "It is important that you be realistic," discourages the patient from feeling hopeful, which is important to patients with any life-threatening diagnosis. Cognitive Level: Application Text Reference: p. 1011 Nursing Process: Implementation NCLEX: Psychosocial Integrity

4. A patient who has been NPO during treatment for nausea and vomiting caused by gastric irritation is to start oral intake. Which of these should the nurse offer to the patient? a. A glass of orange juice b. A bowl of hot chicken broth c. A dish of lemon gelatin d. A cup of coffee with cream

Answer: C Rationale: Clear liquids are usually the first foods started after a patient has been nauseated. Acidic foods such as orange juice, very hot foods, and coffee are poorly tolerated when patients have been nauseated. Cognitive Level: Comprehension Text Reference: pp. 992, 995 Nursing Process: Implementation NCLEX: Physiological Integrity

32. The nurse implements discharge teaching for a patient following a gastroduodenostomy for treatment of a peptic ulcer. Which patient statement indicates that the teaching has been effective? a. "I will need to choose foods that are low in fat and high in carbohydrate." b. "I will try to drink liquids along with my meals." c. "Vitamin injections may be needed to prevent problems with anemia." d. "The surgery has cured my peptic ulcer disease."

Answer: C Rationale: Cobalamin deficiency may occur after partial gastrectomy, and the patient may need to receive cobalamin injections. Foods that have moderate fat and low carbohydrate should be chosen to prevent dumping syndrome. Ingestion of liquids with meals is avoided to prevent dumping syndrome. Peptic ulcer disease (PUD) is a chronic problem, and the patient will need to continue lifestyle changes and perhaps medications to prevent recurrence. Cognitive Level: Application Text Reference: p. 1027 Nursing Process: Evaluation NCLEX: Physiological Integrity

28. The family member of a patient who has suffered massive abdominal trauma in an automobile accident asks the nurse why the patient is receiving famotidine (Pepcid). The nurse will explain that the medication will a. decrease the risk for nausea and vomiting. b. prevent aspiration of gastric contents. c. inhibit the development of stress ulcers. d. lower the chance for H. pylori infection.

Answer: C Rationale: Famotidine is administered to prevent the development of physiologic stress ulcers, which are associated with a major physiologic insult such as massive trauma. Famotidine does not decrease nausea or vomiting, prevent aspiration, or prevent H. pylori infection. Cognitive Level: Application Text Reference: pp. 996, 998, 1017, 1019 Nursing Process: Implementation NCLEX: Physiological Integrity

40. When counseling a patient with a family history of stomach cancer about ways to decrease risk for developing stomach cancer, the nurse will teach the patient to avoid a. chronic use of H2-blocking medications. b. emotionally or physically stressful situations. c. smoked foods such as bacon and ham. d. foods that cause abdominal distension.

Answer: C Rationale: Smoked foods such as bacon, ham, and smoked sausage increase the risk for stomach cancer. Use of H2 blockers, stressful situations, and abdominal distension are not associated with an increased incidence of stomach cancer. Cognitive Level: Application Text Reference: p. 1028 Nursing Process: Implementation NCLEX: Physiological Integrity

37. Which information will be best for the nurse to include when teaching a patient with PUD about dietary management of the disease? a. "You should avoid eating many raw fruits and vegetables." b. "High-protein foods are least likely to cause pain." c. "Avoid foods that cause pain after you eat them." d. "You will need to remain on a bland diet indefinitely."

Answer: C Rationale: The best information is that each individual should choose foods that are not associated with postprandial discomfort. Raw fruits and vegetables may irritate the gastric mucosa, but chewing well seems to decrease this and some patients may tolerate these well. High-protein foods help to neutralize acid, but they also stimulate hydrochloric (HCl) acid secretion and may increase discomfort for some patients. Bland diets may be recommended during an acute exacerbation of PUD, but there is little scientific evidence to support their use. Cognitive Level: Application Text Reference: p. 1021 Nursing Process: Implementation NCLEX: Physiological Integrity

41. The nurse will instruct the patient with GERD who is being discharged after a Stretta procedure that a. acetaminophen (Tylenol) tablets can be used for pain. b. postoperative nausea is an expected symptom. c. gelatin, clear broth, and tea are appropriate foods for the next 24 hours. d. intake and output should be measured and reported to the health care provider.

Answer: C Rationale: The patient should remain on clear liquids for the first 24 hours after the Stretta procedure. Liquid medications, rather than tablets, are used to decrease irritation at the site. The patient is instructed to notify the health care provider if nausea or vomiting occurs. There is no need for the patient to monitor intake and output. Cognitive Level: Application Text Reference: p. 1007 Nursing Process: Implementation NCLEX: Physiological Integrity

12. A patient with recurring heartburn receives a new prescription for esomeprazole (Nexium). In teaching the patient about this medication, the nurse explains that this drug a. reduces the reflux of gastric acid by increasing the rate of gastric emptying. b. coats and protects the lining of the stomach and esophagus from gastric acid. c. treats gastroesophageal reflux disease by decreasing stomach acid production. d. neutralizes stomach acid and provides relief of symptoms in a few minutes.

Answer: C Rationale: The proton pump inhibitors decrease the rate of gastric acid secretion. Promotility drugs such as metoclopramide (Reglan) increase the rate of gastric emptying. Cryoprotective medications such as sucralfate (Carafate) protect the stomach. Antacids neutralize stomach acid and work rapidly. Cognitive Level: Comprehension Text Reference: pp. 998, 1006 Nursing Process: Implementation NCLEX: Physiological Integrity

44. A 22-year-old patient with Escherichia coli O157:H7 food poisoning is admitted to the hospital with bloody diarrhea and dehydration. All of the following orders are received. Which order will the nurse question? a. Infuse lactated Ringer's solution at 250 ml/hr. b. Monitor blood urea nitrogen and creatinine daily. c. Administer loperamide (Imodium) after each stool. d. Provide a clear liquid diet and progress diet as tolerated.

Answer: C Rationale: Use of antidiarrheal agents is avoided with this type of food poisoning. The other orders are appropriate. Cognitive Level: Application Text Reference: pp. 1031, 1033 Nursing Process: Implementation NCLEX: Physiological Integrity

25. A patient who is vomiting bright red blood is admitted to the emergency department. Which assessment should the nurse accomplish first? a. Measuring the quantity of any emesis b. Checking the level of consciousness c. Auscultating the chest for breath sounds d. Taking the blood pressure (BP) and pulse

Answer: D Rationale: The nurse is concerned about blood loss and possible hypovolemic shock in a patient with acute GI bleeding; BP and pulse are the best indicators of these complications. The other information is also important to obtain, but BP and pulse rate are the best indicators for hypoperfusion. Cognitive Level: Application Text Reference: p. 997 Nursing Process: Assessment NCLEX: Physiological Integrity

42. Which nursing diagnosis is appropriate for the home health nurse to use when planning care for a patient who has had a total gastrectomy with an anastomosis of the esophagus to the jejunum for treatment of stomach cancer? a. Chronic pain related to altered GI tract function secondary to the surgery b. Risk for infection related to ongoing need for parenteral nutrition c. Risk for impaired skin integrity related to leakage from jejunostomy tube d. Imbalanced nutrition: less than body requirements related to inability to absorb nutrients

Answer: D Rationale: After this procedure, there will be less surface area for nutrient absorption and vitamins that are normally absorbed in the duodenum will have poor absorption. Chronic pain may occur, but this is due to cancer, not to changes that occur in GI function because of surgery. Parenteral nutrition may be used in the immediate postoperative period but is not needed on an ongoing basis. The patient will not have a jejunostomy tube. Cognitive Level: Application Text Reference: p. 1031 Nursing Process: Diagnosis NCLEX: Physiological Integrity

15. A patient who has recently been experiencing frequent heartburn is seen in the clinic. The nurse will anticipate teaching the patient about a. endoscopy procedures. b. barium swallow. c. radionuclide tests. d. proton pump inhibitors.

Answer: D Rationale: Because diagnostic testing for heartburn that is probably caused by gastroesophageal reflux disease (GERD) is expensive and uncomfortable, proton pump inhibitors are frequently used for a short period as the first step in the diagnosis of GERD. The other tests may be used but are not usually the first step in diagnosis. Cognitive Level: Application Text Reference: p. 1005 Nursing Process: Planning NCLEX: Physiological Integrity

21. Cobalamin injections have been prescribed for a patient with chronic atrophic gastritis. The nurse determines that teaching regarding the injections has been effective when the patient states, a. "These injections will decrease my risk for developing stomach cancer." b. "These injections will increase the hydrochloric acid in my stomach." c. "The cobalamin injections need to be taken until my inflamed stomach heals." d. "The cobalamin injections will prevent me from becoming anemic."

Answer: D Rationale: Cobalamin supplementation prevents the development of pernicious anemia. The incidence of stomach cancer is higher in patients with chronic gastritis, but cobalamin does not reduce the risk for stomach cancer. Chronic gastritis may cause achlorhydria, but cobalamin does not correct this. The loss of intrinsic factor secretion with chronic gastritis is permanent, and the patient will need lifelong supplementation with cobalamin. Cognitive Level: Application Text Reference: p. 1014 Nursing Process: Evaluation NCLEX: Physiological Integrity

19. The nurse will plan to teach the patient with newly diagnosed achalasia that a. drinking fluids with meals should be avoided. b. lying down and resting after meals is recommended. c. a liquid or blenderized diet will be necessary. d. endoscopic procedures may be used for treatment.

Answer: D Rationale: Endoscopic and laparoscopic procedures are the most effective therapy for improving symptoms caused by achalasia. Patients are advised to drink fluid with meals. Keeping the head elevated after eating will improve esophageal emptying. A semisoft diet is recommended to improve esophageal emptying. Cognitive Level: Application Text Reference: p. 1012 Nursing Process: Planning NCLEX: Physiological Integrity

10. The nurse is assessing a patient with gastroesophageal reflux disease (GERD) who is experiencing increasing discomfort. Which patient statement indicates that additional patient education about GERD is needed? a. "I take antacids between meals and at bedtime each night." b. "I quit smoking several years ago, but I still chew a lot of gum." c. "I sleep with the head of the bed elevated on 4-inch blocks." d. "I eat small meals throughout the day and have a bedtime snack."

Answer: D Rationale: GERD is exacerbated by eating late at night, and the nurse should plan to teach the patient to avoid eating at bedtime. The other patient actions are appropriate to control symptoms of GERD. Cognitive Level: Application Text Reference: p. 1005 Nursing Process: Evaluation NCLEX: Physiological Integrity

13. After the nurse teaches a patient with GERD about recommended dietary modifications, which diet choice for a snack 2 hours before bedtime indicates that the teaching has been effective? a. Chocolate pudding b. Glass of low-fat milk c. Peanut butter sandwich d. Cherry gelatin and fruit

Answer: D Rationale: Gelatin and fruit are low fat and will not decrease lower esophageal sphincter (LES) pressure. Foods like chocolate are avoided because they lower LES pressure. Milk products increase gastric acid secretion. High-fat foods such as peanut butter decrease both gastric emptying and LES pressure. Cognitive Level: Application Text Reference: p. 1005 Nursing Process: Evaluation NCLEX: Physiological Integrity

35. A patient who requires daily use of a nonsteroidal antiinflammatory drug (NSAID) for management of severe rheumatoid arthritis has recently developed melena. The nurse will anticipate teaching the patient about a. the use of ranitidine (Zantac) to decrease the risk for peptic ulcers. b. reasons for using corticosteroids to treat the arthritis. c. substitution of acetaminophen (Tylenol) for the NSAID. d. the benefits of misoprostol (Cytotec) in protecting the GI mucosa.

Answer: D Rationale: Misoprostol, a prostaglandin analog, is the only drug approved in the United States for preventing gastric ulcers induced by NSAIDs. Ranitidine does increase pH but is not approved for prevention of ulcers in patients chronically taking NSAIDs. Corticosteroids increase risk for ulcer development and will not be substituted for NSAIDs for this patient. Acetaminophen will not be effective in treating the patient's rheumatoid arthritis. Cognitive Level: Application Text Reference: p. 1021 Nursing Process: Planning NCLEX: Physiological Integrity

3. Which of these nursing actions should the RN working in the emergency department delegate to a nursing assistant who is helping with the care of a patient who has been admitted with nausea and vomiting? a. Assess for signs of dehydration. b. Ask the patient what precipitated the nausea. c. Auscultate the bowel sounds. d. Assist the patient with oral care after vomiting.

Answer: D Rationale: Oral care is included in nursing assistant education and scope of practice. The other actions are all assessments that require more education and a higher scope of nursing practice. Cognitive Level: Application Text Reference: pp. 992-995 Nursing Process: Implementation NCLEX: Physiological Integrity

A 55-year-old patient admitted with an abrupt onset of jaundice and nausea has abnormal liver function studies but serologic testing is negative for viral causes of hepatitis. Which question by the nurse is most appropriate? a. "Is there any history of IV drug use?" b. "Do you use any over-the-counter drugs?" c. "Are you taking corticosteroids for any reason?" d. "Have you recently traveled to a foreign country?"

B

The nurse recognizes that teaching a 44-year-old woman following a laparoscopic cholecystectomy has been effective when the patient states which of the following? a. "I can expect yellow-green drainage from the incision for a few days." b. "I can remove the bandages on my incisions tomorrow and take a shower." c. "I should plan to limit my activities and not return to work for 4 to 6 weeks." d. "I will always need to maintain a low-fat diet since I no longer have a gallbladder."

B

Which information given by a 70-year-old patient during a health history indicates to the nurse that the patient should be screened for hepatitis C? a. The patient had a blood transfusion in 2005. b. The patient used IV drugs about 20 years ago. c. The patient frequently eats in fast-food restaurants. d. The patient traveled to a country with poor sanitation.

B

Which topic is most important to include in patient teaching for a 41-year-old patient diagnosed with early alcoholic cirrhosis? a. Maintaining good nutrition b. Avoiding alcohol ingestion c. Taking lactulose (Cephulac) d. Using vitamin B supplements

B

A female patient expresses her concern about becoming pregnant while her partner is on ribavirin therapy for chronic hepatitis C. What should the nurse advise the patient? a. She can plan pregnancy now. b. She should avoid getting pregnant now. c. She should not get pregnant with this partner ever. d. She should avoid any sexual intercourse after conception.

B Any woman who is on ribavirin or whose male partner is on is on ribavirin should avoid pregnancy during treatment. The pregnancy can be planned after the treatment is complete. She can get pregnant with this partner, but not while on treatment. Avoiding intercourse after conception is not necessary. Text Reference - p. 1013

What are the precautions that nurses and hospital staff should follow while handling patients suffering from hepatitis infections? Select all that apply. a. The patient must be in a private room, and door should be closed. b. Dispose of the needles and syringes used on the patient carefully. c. Wear gloves while handling articles contaminated by urine or feces. d. Always wear a mask, gown, and gloves when entering the patient's room. e. Follow infection control precautions while injecting the patient, and avoid getting pricked by the used needle.

B, C, E Hepatitis A spreads through the fecal-oral route, and hepatitis B spreads through blood. Hence the virus can spread through needles and syringes used by the patient. Also, the virus can spread while the nurse is handling the urine or fecal material of the patient; hence it is necessary to wear gloves. Hepatitis does not spread through air; hence a mask is not required. A private room is required in respiratory diseases, not in hepatitis. Test-Taking Tip: Study wisely, not hard. Use study strategies to save time and be able to get a good night's sleep the night before your exam. Cramming is not smart, and it is hard work that increases stress while reducing learning. When you cram, your mind is more likely to go blank during a test. When you cram, the information is in your short-term memory so you will need to relearn it before a comprehensive exam. Relearning takes more time. The stress caused by cramming may interfere with your sleep. Your brain needs sleep to function at its best. Text Reference - p. 1014

The nurse provides discharge instructions to a patient with newly diagnosed cirrhosis. Which statement made by the patient indicates the need for further teaching? a. "I should take frequent rest periods." b. "I can eat anything that appeals to me." c. "I can do without my glass of wine with dinner." d. "I should take only medications that have been prescribed."

B. Even though a low-protein diet has been questioned in the treatment of patients with cirrhosis, it remains in use. In light of this, it is incorrect for the patient to say that he may eat anything. Patients with cirrhosis must also avoid alcohol. Frequent rest and limitation of medications to those that have been prescribed are appropriate resolutions in a newly diagnosed case of cirrhosis and therefore do not indicate the need for further teaching. Text Reference - p. 1022

Laboratory test results that the nurse would expect to find in a patient with cirrhosis include a. serum albumin 7.0 g/dL b. bilirubin total 3.2 mg/dL c. serum cholesterok 260 mg/dL d. aspartate aminotransferase (AST) 6.0 U/L

B- Serum bilirubin, both direct and indirect, would be expected to be increased in cirrhosis. Serum albumin and cholesterol are decreased, and liver enzymes, such as AST and ALT, are elevated

A patient with advanced cirrhosis has a nursing diagnosis of imbalanced nutrition: less than body requirements r/t anorexia and inadequate food intake. An appropriate midday snack for the patient would be a. peanut butter and salt free crackers b. a fresh tomato sandwich with salt free butter c. popcorn with salt free butter and herbal seasoning d. canned chicken noodle soup with low protein bread

B- The patient with advanced, complicated cirrhosis requires a high calore, high carbohydrate diet with moderate to low fat. Patients with cirrhosis are at risk for edema and ascites and their sodium intake should be limited. The tomato sandwich with salt free butter best meets these requirements. Rough foods, such as popcorn, may irritate the esophagus and stomach and lead to bleeding. Peanut butter is high in sodium and fat, and canned chicken noodle soup is very high in sodium

The nurse identifies a need for further teaching when the patient with hepatitis B states, a. I should avoid alcohol completely for as long as a year b. I must avoid all physical contact with my family until the jaundice is gone c. I should use a condom to prevent spread of the disease to my sexual partners d. I will need to rest several times a day, gradually increasing my activity as I tolerate it.

B- The patient with hep B is infectious for 4 to 6 months, and precautions to prevent transmission through percutaneous and sexual contact should be maintained until tests for HBsAg are negative. Close contact does not have to be avoided, but close contacts of the patient should be vaccinated. Alcohol should not be used for at least a year, and rest with increasing activity during convalescence is recommended

The patient with advanced cirrhosis asks why his or her skin is so yellow. The nurse's response is based on the knowledge that: a. Decreased peristalsis in the gastrointestinal tract contributes to a buildup of bile salts. b. Jaundice results from the body's inability to conjugate and excrete bilirubin. c. A lack of clotting factors promotes the collection of blood under the skin surface. d. Decreased colloidal oncotic pressure from hypoalbuminemia causes the yellowish skin discoloration.

B. Jaundice results from the functional derangement of liver cells and compression of bile ducts by connective tissue overgrowth. Jaundice occurs as a result of the decreased ability to conjugate and excrete bilirubin Jaundice is not caused by a build-up of bile salts, a lack of clotting factors, or decreased colloidal oncotic pressure. Test-Taking Tip: The most reliable way to ensure that you select the correct response to a multiple-choice question is to recall it. Depend on your learning and memory to furnish the answer to the question. To do this, read the stem, and then stop! Do not look at the response options yet. Try to recall what you know and, based on this, what you would give as the answer. After you have taken a few seconds to do this, then look at all of the choices and select the one that most nearly matches the answer you recalled. It is important that you consider all the choices and not just choose the first option that seems to fit the answer you recall. Remember the distractors. The second choice may look okay, but the fourth choice may be worded in a way that makes it a slightly better choice. If you do not weigh all the choices, you are not maximizing your chances of correctly answering each question. Text Reference - p. 1009

A patient has an increased ammonia level associated with hepatic encephalopathy. What assessment finding does the nurse expect? a. Aphasia b. Asterixis c. Hyperactivity d. Acute dementia

B. Asterixis is a twitching spasm of the hands and wrists seen in patients with increased ammonia levels in conditions such as hepatic encephalopathy. Aphasia, hyperactivity, and acute dementia are manifestations not associated with hepatic encephalopathy. Besides asterixis, an increased serum ammonia level causes sedation and confusion that progress to a comatose state. Text Reference - p. 1021

A patient has been admitted with diabetes mellitus, malnutrition, osteomyelitis, and alcohol abuse. Laboratory results are significant for an alanine aminotransferase (ALT) of 198 IU/L and aspartate transaminase (AST) of 224 IU/L. Which diagnosis does the nurse attribute these findings to? a. Diabetes mellitus b. Alcohol abuse c. Malnutrition d. Osteomyelitis

B. In the patient with alcohol abuse, liver disease could develop as a complication, increasing the liver function tests above the normal levels. The normal ALT range is 7 to 56 IU/L and the normal AST range is 5 to 40 IU/L. Diabetes would result in elevated blood sugar levels. Malnutrition would be evidenced by low protein levels. Osteomyelitis is an infection of the bone, which would result in an elevated white blood cell count. Text Reference - p. 1015

2. A nurse is caring for a client who has a new diagnosis of hepatitis C. Which of the following is an expected laboratory finding? A. Presence of immunoglobin G antibodies (IgG) B. Presence of enzyme immunoassay (EIA) C. Aspartate aminotransferase (AST) 35 units/L D. Alanine aminotransferase (ALT) 15 IU/L

B. CORRECT: The presence of enzyme immunoassay is an expected laboratory finding in a client who has a new diagnosis of hepatitis C.

The nurse is caring for a 55-year-old man patient with acute pancreatitis resulting from gallstones. Which clinical manifestation would the nurse expect the patient to exhibit? A. Hematochezia B. Left upper abdominal pain C. Ascites and peripheral edema D. Temperature over 102o F (38.9o C)

B. Left upper abdominal pain Abdominal pain (usually in the left upper quadrant) is the predominant manifestation of acute pancreatitis. Other manifestations of acute pancreatitis include nausea and vomiting, low-grade fever, leukocytosis, hypotension, tachycardia, and jaundice. Abdominal tenderness with muscle guarding is common. Bowel sounds may be decreased or absent. Ileus may occur and causes marked abdominal distention. Areas of cyanosis or greenish to yellow-brown discoloration of the abdominal wall may occur. Other areas of ecchymoses are the flanks (Grey Turner's spots or sign, a bluish flank discoloration) and the periumbilical area (Cullen's sign, a bluish periumbilical discoloration).

Which action should the nurse take to evaluate treatment effectiveness for a patient who has hepatic encephalopathy? a. Request that the patient stand on one foot. b. Ask the patient to extend both arms forward. c. Request that the patient walk with eyes closed. d. Ask the patient to perform the Valsalva maneuver.

B:Extending the arms allows the nurse to check for asterixis, a classic sign of hepatic encephalopathy

In discussing long term management with the patient with alcoholic cirrhosis, the nurse advises the patient that a. a daily exercise regimen is important to increase the blood flow through the liver b. cirrhosis can be reversed if the patient follows a regimen of proper rest and nutrition c. abstinence from alcohol is the most important factor in improvement of the patient's condition d. the only over the counter analgesic that should be used for minor aches and pains is acetaminophen

C- Abstinence from alcohol is very important in alcoholic cirrhosis and may result in improvement if started when liver damage is reduced by rest and nutrition, most changes in the liver cannot be reversed. Exercise does not promote portal circulation, and very moderate exercise is recommended. Acetaminophen should not be used by the patient with liver disease because it is potentially hepatotoxic.

During the treatment of the patient with bleeding esophageal varices, it is most important that the nurse a. prepare the patient for immediate portal shunting surgery b. perform guaiac testing on all stools to detect occult blood c. maintain the patient's airway and prevent aspiration of blood d. monitor for the cardiac effects of IV vasopressin and nitroglycerin

C- Bleeding esophageal varices are a medical emergency. During an episode of bleeding, management of the airway and prevention of aspiration of blood are critical factors. Occult blood as well as fresh blood from the GI tract would be expected and is not tested. Vasopressin causes vasoconstriction, decreased HR, and decreased coronary blood flow; nitroglycerin is given with the vasopressin to counter these side effects. Portal shunting surgery is performed for esophageal varices but not during an acute hemorrhage

When caring for a patient with autoimmune hepatitis, the nurse recognizes that, unlike viral hepatitis, the patient a. does not manifest hepatomegaly or jaundice b. experiences less liver inflammation and damage c. is treated with corticosteroids or other immunosuppressant agents d. is usually an older adult who has used a wide variety of prescription and over the counter drugs

C- Immunosuppressive agents are indicated i hepatitis associated with immune disorders to decrease liver damage caused by autoantibodies. Autoimmune hepatitis is similar to viral hepatitis in presenting signs and symptoms and may become chronic and lead to cirrhosis.

During the incubation period of viral hepatitis, the nurse would expect the patient to report a. pruritus and malaise b. dark urine and easy fatigability c. anorexia and right upper quadrant discomfort d. constipation or diarrhea with light colored stools

C- Incubation symptoms occur before the onset of jaundice and include a variety of GI symptoms as well as discomfort and heaviness in the upper right quadrant of the abdomen. Pruritus, dark urine, and light colored stools occur with the onset of jaundice in the acute phase.

A patient with cancer that has metastasized to the liver manifests symptoms of fluid retention, including edema and ascites. To determine the effectiveness of the diuretic therapy that has been prescribed, what should the nurse assess? a. Breath sounds b. Bowel sounds c. Abdominal girth d. Recent blood work

C. Daily measurement of the abdominal girth provides a direct indication of ascitic fluid increase or decrease. Breath and bowel sounds are usually not affected by liver metastasis until the late stages, when fluid overload and multisystem organ involvement occur. Reviewing the results of the most recent blood work will not show direct measurement of the effectiveness of diuretic therapy. Text Reference - p. 1019

A patient admitted to the hospital with cirrhosis of the liver suddenly starts vomiting blood. What is the priority action that the nurse should take in this situation? a. Send for endoscopic variceal ligation. b. Give propronalol orally. c. Stabilize the patient and manage the airway. d. Check for signs of cirrhosis

C. Individuals with cirrhosis of the liver are at risk of bleeding from esophageal and gastric varices. Hematemesis in the patient with cirrhosis of the liver is likely to be variceal bleeding. In this case, the nurse should first stabilize the patient and manage the airway. Once the patient is stable, other steps in treatment can be initiated, such as assessing further and administering necessary medications. Text Reference - p. 1025

The nurse evaluates the effectiveness of a paracentesis in a patient who has ascites. Which measurement is most important for the nurse to note? a. Cardiac output b. Blood pressure c. Abdominal girth d. Intake and output

C. Paracentesis involves the removal of fluid from the abdominal cavity. A large-bore needle connected to tubing is inserted by the health care provider into the distended abdomen. The other end of the tubing also has a large-bore needle, which is inserted into a vacuum bottle. The vacuum bottle is then held below the level of the abdomen, facilitating gravity-flowed removal of the ascites. Several bottles of fluid can be removed, with the result measured by reduction in abdominal girth. Cardiac output may improve after paracentesis, but it is unlikely that this measurement needs to be recorded. Paracentesis has no major effect on blood pressure. Likewise, intake and output continue to be monitored to account for the paracentesis fluid but these are not as informative as abdominal girth. Text Reference - p. 1022

The nurse recalls that hepatic coma results primarily from accumulation of which substance in the blood? a. Sodium b. Calcium c. Ammonia d. Potassium

C. A high ammonia level in the blood is a late manifestation of liver failure that results in hepatic coma, causing neurologic dysfunction and brain damage. Sodium, calcium, and potassium are not directly affected by liver dysfunction or hepatic coma. Text Reference - p. 1021

A patient with hepatitis A asks whether other family members are at risk for "catching" the disease. The nurse's response will be based on the knowledge that hepatitis A is transmitted primarily: a. During sexual intercourse b. By contact with infected body secretions c. Through fecal contamination of food or water d. Through kissing that involves contact with mucous membranes

C. Hepatitis A is primarily transmitted through ingestion of organisms on fecally contaminated hands, food, or water. Care should be taken in the handling of food and water, as well as contaminated items such as bed linens, bedpans, and toilets. Hand hygiene and personal protective equipment such as gloves are important in preventing the spread of infection for hospital personnel. In the home, hand hygiene and good personal hygiene are important in decreasing the risk of transmission. Sexual intercourse, contact with infected body secretions, and contact through mucous membranes all present higher risk for hepatitis B and C than for hepatitis A. Text Reference - p. 1007

The nurse provides discharge instructions for a 64-year-old woman with ascites and peripheral edema related to cirrhosis. Which statement, if made by the patient, indicates teaching was effective? A. "It is safe to take acetaminophen up to four times a day for pain." B. "Lactulose (Cephulac) should be taken every day to prevent constipation." C. "Herbs and other spices should be used to season my foods instead of salt." D. "I will eat foods high in potassium while taking spironolactone (Aldactone)."

C. "Herbs and other spices should be used to season my foods instead of salt." A low-sodium diet is indicated for the patient with ascites and edema related to cirrhosis. Table salt is a well-known source of sodium and should be avoided. Alternatives to salt to season foods include the use of seasonings such as garlic, parsley, onion, lemon juice, and spices. Pain medications such as acetaminophen, aspirin, and ibuprofen should be avoided as these medications may be toxic to the liver. The patient should avoid potentially hepatotoxic over-the-counter drugs (e.g., acetaminophen) because the diseased liver is unable to metabolize these drugs. Spironolactone is a potassium-sparing diuretic. Lactulose results in the acidification of feces in bowel and trapping of ammonia, causing its elimination in feces.

The nurse instructs a 50-year-old woman about cholestyramine to reduce pruritis caused by gallbladder disease. Which statement by the patient to the nurse indicates she understands the instructions? A. "This medication will help me digest fats and fat-soluble vitamins." B. "I will apply the medicated lotion sparingly to the areas where I itch." C. "The medication is a powder and needs to be mixed with milk or juice." D. "I should take this medication on an empty stomach at the same time each day."

C. "The medication is a powder and needs to be mixed with milk or juice." For treatment of pruritus, cholestyramine may provide relief. This is a resin that binds bile salts in the intestine, increasing their excretion in the feces. Cholestyramine is in powder form and should be mixed with milk or juice before oral administration.

A patient with cholelithiasis needs to have the gallbladder removed. Which patient assessment is a contraindication for a cholecystectomy? A. Low-grade fever of 100° F and dehydration B. Abscess in the right upper quadrant of the abdomen C. Activated partial thromboplastin time (aPTT) of 54 seconds D. Multiple obstructions in the cystic and common bile duct

C. Activated partial thromboplastin time (aPTT) of 54 seconds An aPTT of 54 seconds is above normal and indicates insufficient clotting ability. If the patient had surgery, significant bleeding complications postoperatively are very likely. Fluids can be given to eliminate the dehydration; the abscess can be assessed, and the obstructions in the cystic and common bile duct would be relieved with the cholecystectomy.

The condition of a patient who has cirrhosis of the liver has deteriorated. Which diagnostic study would help determine if the patient has developed liver cancer? A. Serum α-fetoprotein level B. Ventilation/perfusion scan C. Hepatic structure ultrasound D. Abdominal girth measurement

C. Hepatic structure ultrasound Correct Hepatic structure ultrasound, CT, and MRI are used to screen and diagnose liver cancer. Serum α-fetoprotein level may be elevated with liver cancer or other liver problems. Ventilation/perfusion scans do not diagnose liver cancer. Abdominal girth measurement would not differentiate between cirrhosis and liver cancer.

A 49-year-old female patient with cirrhosis and esophageal varices has a new prescription for propranolol (Inderal). Which finding is the best indicator that the medication has been effective? a. The patient reports no chest pain. b. Blood pressure is 140/90 mm Hg. c. Stools test negative for occult blood. d. The apical pulse rate is 68 beats/minute.

C: the purpose of Beta-blocker therapy for patients with esophageal varices is to decrease the risk for bleeding from esophageal varices,

A patient with hepatitis A is in the acute phase. The nurse plans care for the patient based on the knowledge that a. pruritus is a common problem with jaundice in this phase. b. the patient is most likely to transmit the disease during this phase. c. gastrointestinal symptoms are not as severe in hepatitis A as they are in hepatitis B. d. extrahepatic manifestations of glomerulonephritis and polyarteritis are common in this phase.

Correct answer: a Rationale: The acute phase of jaundice may be icteric or anicteric. Jaundice results when bilirubin diffuses into the tissues. Pruritus sometimes accompanies jaundice. Pruritus is the result of an accumulation of bile salts beneath the skin.

which of the following conditions most places a pt. with cirrhosis at risk for bleeding? a. encephalopathy b. low vitamin K c. elevated liver enzymes d. hepatorenal syndrome

low vitamin K: vitamin k is needed for blood clotting, so a low level increases the risk of bleeding

The patient with advanced cirrhosis asks why his abdomen is so swollen. The nurse's response is based on the knowledge that a. a lack of clotting factors promotes the collection of blood in the abdominal cavity. b. portal hypertension and hypoalbuminemia cause a fluid shift into the peritoneal space. c. decreased peristalsis in the GI tract contributes to gas formation and distention of the bowel. d. bile salts in the blood irritate the peritoneal membranes, causing edema and pocketing of fluid.

Correct answer: b Rationale: Ascites is the accumulation of serous fluid in the peritoneal or abdominal cavity and is a common manifestation of cirrhosis. With portal hypertension, proteins shift from the blood vessels through the larger pores of the sinusoids (capillaries) into the lymph space. When the lymphatic system is unable to carry off the excess proteins and water, those substances leak through the liver capsule into the peritoneal cavity. Osmotic pressure of the proteins pulls additional fluid into the peritoneal cavity. A second mechanism of ascites formation is hypoalbuminemia, which results from the inability of the liver to synthesize albumin. Hypoalbuminemia results in decreased colloidal oncotic pressure. A third mechanism is hyperaldosteronism, which occurs when aldosterone is not metabolized by damaged hepatocytes. The increased level of aldosterone causes increases in sodium reabsorption by the renal tubules. Sodium retention and an increase in antidiuretic hormone levels cause additional water retention.

A patient with acute hepatitis B is being discharged in 2 days. In the discharge teaching plan the nurse should include instructions to a. avoid alcohol for the first 3 weeks. b. use a condom during sexual intercourse. c. have family members get an injection of immunoglobulin. d. follow a low-protein, moderate-carbohydrate, moderate-fat diet.

Correct answer: b Rationale: Hepatitis B virus may be transmitted by mucosal exposure to infected blood, blood products, or other body fluids (e.g., semen, vaginal secretions, saliva). Hepatitis B is a sexually transmitted disease that is acquired through unprotected sex with an infected person. Condom use should be taught to patients to prevent transmission of hepatitis B.

A patient has been told that she has elevated liver enzymes caused by nonalcoholic fatty liver disease (NAFLD). The nursing teaching plan should include a. having genetic testing done. b. recommending a heart-healthy diet. c. the necessity to reduce weight rapidly. d. avoiding alcohol until liver enzymes return to normal.

Correct answer: b Rationale: Nonalcoholic fatty liver disease (NAFLD) can progress to liver cirrhosis. There is no definitive treatment, and therapy is directed at reduction of risk factors, which include treatment of diabetes, reduction in body weight, and elimination of harmful medications. For patients who are overweight, weight reduction is important. Weight loss improves insulin sensitivity and reduces liver enzyme levels. No specific dietary therapy is recommended. However, a heart-healthy diet as recommended by the American Heart Association is appropriate.

A patient with cirrhosis that is refractory to other treatments for esophageal varices undergoes a peritoneovenous shunt. As a result of this procedure, the nurse would expect the patient to experience a. an improved survival rate b. decreased serum ammonia levels c. improved metabolism of nutrients d. improved hemodynamic function and renal perfusion

D- By shunting fluid sequestered in the peritoneum into the venous system, pressur eon esophageal veins is decreased, and more volume is returned to the circulation, improving CO and renal perfusion. However, because ammonia is diverted past the liver, hepatic encephalopathy continues. These procedures do not prolong life or promote liver function.

When teaching the male patient with acute hepatitis C (HCV), the patient demonstrates understanding when the patient makes which statement? a. "I will use care when kissing my wife to prevent giving it to her." b. "I will need to take adofevir to prevent chronic HCV." c. "Now that I have had HCV, I will have immunity and not get it again." d. "I will need to be checked for chronic HCV and other liver problems."

D The majority of patients who acquire HCV usually develop chronic infection, which may lead to cirrhosis or liver cancer. HCV is not transmitted via saliva, but percutaneously and via high-risk sexual activity exposure. The treatment for acute viral hepatitis focuses on resting the body and adequate nutrition for liver regeneration. Adofevir is taken for severe hepatitis B (HBV) with liver failure. Chronic HCV is treated with pegylated interferon with ribavirin. Immunity with HCV does not occur as it does with hepatitis A virus (HAV) and HBV, so the patient may be reinfected with another type of HCV. Text Reference - p. 1015

A patient with hepatitis A infection is being discharged from the hospital. What is the most important instruction that the nurse should include in the discharge teaching? a. Do not share razors or toothbrushes. b. Isolate the patient from other family members. c. Take acetaminophen every four hours if fever persists. d. Wash hands carefully after bowel movements

D The mode of transmission of hepatitis A infection is the fecal-oral route. Therefore, it is very important to maintain personal and environmental hygiene. The nurse should teach the patient and the family members about careful hand washing immediately after bowel movements and before eating to prevent outbreaks of hepatitis A viral infection. Not sharing toothbrushes and razors is a concern for the prevention of hepatitis B and C, because they are transferred through blood contact. There is no need to isolate the patient with hepatitis A unless he or she is incontinent or maintains poor personal hygiene. Acetaminophen may cause liver damage and should be avoided in hepatic viral infection. Test-Taking Tip: After choosing an answer, go back and reread the question stem along with your chosen answer. Does it fit correctly? The choice that grammatically fits the stem and contains the correct information is the best choice. Text Reference - p. 1014

The family members of a patient with hepatitis A ask if there is anything that will prevent them from developing the disease. The best response by the nurse is a. "no immunization is available for hepatitis A, nor are you likely to get the disease" b. "only individuals who have had sexual contact with the patient should receive immunization" c. "all family members should receive the hepatitis A vaccine to prevent or modify the infection" d. "those who have had household or close contact with the patient should receive immune globulin"

D- Individuals who have been exposed to hepatitis A through household contact or foodborne outbreaks should be given immune globulin within 1 to 2 weeks of exposure to prevent or modify the illness. Hep A vaccine is used to provide pre-exposure immunity to the virus and is indicated for individuals at high risk for hep A exposure. Although hep A can be spread by sexual contact, the risk is higher for transmission with the oral-fecal route.

The systemic effects of viral hepatitis are caused primarily by a. cholestasis b. impaired portal circulation c. toxins produced by the infected liver d. activation of the complement system by antigen-antibody complexes

D- The systemic manifestations of rash, angioedema, arthritis, fever, and malaise in viral hepatitis are caused by the activation o the complement system by circulating immune complexes. Liver manifestations include jaundice from hepatic cell damage and cholestasis as well as anorexia perhaps caused by toxins produced by the damaged liver. Impaired portal circulation usually does not occur in uncomplicated viral hepatitis but would be a liver manifestation

The patient with a history of lung cancer and hepatitis C has developed liver failure and is considering liver transplantation. After the comprehensive evaluation, the nurse knows that which factor discovered may be a contraindication for liver transplantation? The patient has completed a college education. The patient has been able to stop smoking cigarettes. The patient has well controlled type 1 diabetes mellitus. The chest x-ray showed another lung cancer lesion

D. Contraindications for liver transplant include severe extrahepatic disease, advanced hepatocellular carcinoma or other cancer, ongoing drug or alcohol abuse, and the inability to comprehend or comply with the rigorous post-transplant course. It does not matter if the patient has a college education. The fact that the patient has quit smoking is not a contraindication for liver transplant. The patient is a well-controlled diabetic, which is not a contraindication.

The nurse is admitting a patient with cirrhosis. The nurse checks the patient's history for which most frequent risk factor associated with cirrhosis? a. Polypharmacy b. Intravenous drug abuse c. Hepatitis A d. Alcohol abuse

D. Cirrhosis is highly correlated with alcohol abuse. Polypharmacy, drug abuse, and hepatitis A are not linked to cirrhosis. Text Reference - p. 1017

A patient with a 3-year history of liver cirrhosis is hospitalized for treatment of recently diagnosed esophageal varices. What is the most important information for the nurse to include in the teaching plan for this patient? a. Decrease fluid intake to avoid ascites. b. Eat foods quickly so they do not get cold and cause distress. c. Avoid exercise because it may cause bleeding of the varices. d. Avoid straining during defecation to keep venous pressure low

D. Straining during a bowel movement increases venous pressure and could cause rupture of the varices. Fluid restrictions may be a recommendation for ascites but are not directly associated with esophageal varices. If the patient is able to eat, meals should be soft or liquid, and the patient should be instructed to eat slowly and avoid extremes in food temperature to prevent irritation. Excessive exercise and activity should be avoided in a patient with esophageal varices to prevent hypertension, however, avoiding straining and other activities that cause the Valsalva maneuver is still a higher-priority recommendation. Text Reference - p. 1022

The family of a patient newly diagnosed with hepatitis A asks the nurse what they can do to prevent becoming ill themselves. Which response by the nurse is most appropriate? A. "The hepatitis vaccine will provide immunity from this exposure and future exposures." B. "I am afraid there is nothing you can do since the patient was infectious before admission." C. "You will need to be tested first to make sure you don't have the virus before we can treat you." D. "An injection of immunoglobulin will need to be given to prevent or minimize the effects from this exposure."

D. "An injection of immunoglobulin will need to be given to prevent or minimize the effects from this exposure." Immunoglobulin provides temporary (1-2 months) passive immunity and is effective for preventing hepatitis A if given within 2 weeks after exposure. It may not prevent infection in all persons, but it will at least modify the illness to a subclinical infection. The hepatitis vaccine is only used for preexposure prophylaxis.

5. A nurse is completing nutrition teaching for a client who has pancreatitis. Which of the following statements by the client requires further teaching? A. "I plan to eat small, frequent meals." B. "I will eat easy-to-digest foods with limited spice." C. "I will use skim milk when cooking." D. "I plan to drink regular cola."

D. CORRECT: Caffeine-free beverages are recommended for the client who has pancreatitis. Regular cola contains caffeine.

3. A nurse is completing an admission assessment of a client who has pancreatitis. Which of the following is an expected finding? A. Pain in right upper quadrant radiating to right shoulder B. Report of pain being worse when sitting upright C. Pain relieved with defecation D. Epigastric pain radiating to left shoulder

D. CORRECT: A client who has pancreatitis will report severe, boring epigastric pain that radiates to the back, left flank, or left shoulder.

1. A nurse on a medical-surgical unit is admitting a client who has hepatitis B with ascites. Which of the following actions should the nurse include in the plan of care? A. Initiate contact precautions. B. Weigh client weekly. C. Measure abdominal girth 7.5 cm (3 in) above the umbilicus. D. Provide a high-calorie, high-carbohydrate diet.

D. CORRECT: A high-calorie, high-carbohydrate diet is recommended for clients who have hepatitis B.

1. A nurse is completing the admission assessment of a client who has acute pancreatitis. Which of the following findings is the priority to be reported to the provider? A. History of cholelithiasis B. Serum amylase levels three times greater than the expected value C. Client report of severe pain radiating to the back that is rated at an "8" D. Hand spasms present when blood pressure is checked

D. CORRECT: The greatest risk to the client is hypocalcemia due to the risk of cardiac dysrhythmia. Hand spasms when taking a blood pressure is an indication of hypocalcemia and is the priority finding to report to the provider

4. A nurse is reviewing a new prescription for ursodiol (Ursodeoxycholic Acid) with a client who has cholelithiasis. Which of the following should be included in the teaching? A. This medication reduces biliary spasms. B. This medication reduces inflammation in the biliary tract. C. This medication dilates the bile duct to promote passage of bile. D. This medication dissolves gall stones.

D. CORRECT: Ursodiol is a bile acid that gradually dissolves cholesterol-based gall stones.

The patient with suspected pancreatic cancer is having many diagnostic studies done. Which one can be used to establish the diagnosis of pancreatic adenocarcinoma and for monitoring the response to treatment? A. Spiral CT scan B. A PET/CT scan C. Abdominal ultrasound D. Cancer-associated antigen 19-9

D. Cancer-associated antigen 19-9 The cancer-associated antigen 19-9 (CA 19-9) is the tumor marker used for the diagnosis of pancreatic adenocarcinoma and for monitoring the response to treatment. Although a spiral CT scan may be the initial study done and provides information on metastasis and vascular involvement, this test and the PET/CT scan or abdominal ultrasound do not provide additional information.

The health care provider orders lactulose for a patient with hepatic encephalopathy. The nurse will monitor for effectiveness of this medication for this patient by assessing what? A. Relief of constipation B. Relief of abdominal pain C. Decreased liver enzymes D. Decreased ammonia levels

D. Decreased ammonia levels. Hepatic encephalopathy is a complication of liver disease and is associated with elevated serum ammonia levels. Lactulose traps ammonia in the intestinal tract. Its laxative effect then expels the ammonia from the colon, resulting in decreased serum ammonia levels and correction of hepatic encephalopathy

A patient who has hepatitis B surface antigen (HBsAg) in the serum is being discharged with pain medication after knee surgery. Which medication order should the nurse question because it is most likely to cause hepatic complications? A. Tramadol (Ultram) B. Hydromorphone (Dilaudid) C. Oxycodone with aspirin (Percodan) D. Hydrocodone with acetaminophen (Vicodin)

D. Hydrocodone with acetaminophen (Vicodin) The analgesic with acetaminophen should be questioned because this patient is a chronic carrier of hepatitis B and is likely to have impaired liver function. Acetaminophen is not suitable for this patient because it is converted to a toxic metabolite in the liver after absorption, increasing the risk of hepatocellular damage.

When planning care for a patient with cirrhosis, the nurse will give highest priority to which nursing diagnosis? A. Impaired skin integrity related to edema, ascites, and pruritus B. Imbalanced nutrition: less than body requirements related to anorexia C. Excess fluid volume related to portal hypertension and hyperaldosteronism D. Ineffective breathing pattern related to pressure on diaphragm and reduced lung volume

D. Ineffective breathing pattern related to pressure on diaphragm and reduced lung volume Although all of these nursing diagnoses are appropriate and important in the care of a patient with cirrhosis, airway and breathing are always the highest priorities.

Which assessment finding would the nurse need to report most quickly to the health care provider regarding a patient with acute pancreatitis? a. Nausea and vomiting b. Hypotonic bowel sounds c. Abdominal tenderness and guarding d. Muscle twitching and finger numbness

D: Muscle twitching and finger numbness indicate hypocalcemia,

Nursing management of the patient with acute pancreatitis includes (select all that apply) a. checking for signs of hypocalcemia. b. providing a diet low in carbohydrates. c. giving insulin based on a sliding scale. d. observing stools for signs of steatorrhea. e. monitoring for infection, particularly respiratory tract infection.

a, e Rationale: During the acute phase, it is important to monitor vital signs. Hemodynamic stability may be compromised by hypotension, fever, and tachypnea. Intravenous fluids are ordered, and the response to therapy is monitored. Fluid and electrolyte balances are closely monitored. Frequent vomiting, along with gastric suction, may result in decreased levels of chloride, sodium, and potassium. Because hypocalcemia can occur in acute pancreatitis, the nurse should observe for symptoms of tetany, such as jerking, irritability, and muscular twitching. Numbness or tingling around the lips and in the fingers is an early indicator of hypocalcemia. The patient should be assessed for Chvostek's sign or Trousseau's sign. A patient with acute pancreatitis should be observed for fever and other manifestations of infection. Respiratory infections are common because the retroperitoneal fluid raises the diaphragm, which causes the patient to take shallow, guarded abdominal breaths.

Describe the pathophysiologic changes of cirrhosis that cause the following a. Portal hypertension b. Esophageal varices

a. scarring and nodular changes in liver lead to compression of the veins and sinusoids, causing resistance of blood flow through the liver from the portal vein b. development of collateral channels of circulation in inelastic, fragile esophageal veins as a result of portal hypertension

in planning care for the newly admitted pt with acute pancreatitis, which pt outcome should receive highest priority? a. pt expresses satisfaction with pain control b. pt verbalizes understanding of medications for home c. pt increases activity intolerance d. pt maintains normal bowl function

a: because pancreatitis is very painful, pt satisfaction with pain control is a priority

a 44 y/o client has cirrhosis of the liver with sever ascites. the nurse plans for the following intervention? 1. apply prolonged pressure to injection sites 2. administer aspirin for headache as needed 3. measure abdominal girth weekly 4. provide a low protein, high carb diet

apply prolonged pressure to injection sites: the client may have a tendency to bleed easily bc clotting factors are formed in the liver

the male client dx with chronic pancreatitis calls and reports to the clinic nurse he has been having a lot of gas along with frothy and very foul smelling stools. which intervention should the nurse implement? 1. explain this is common for chronic pancreatitis 2. ask the client to bring in a stool specimen to the clinic 3. arrange an appointment with the hcp for today 4. discuss the need to decrease fat in the diet so this wont happen

arrange an appointment with the hcp for today: steatorrhea (fatty, frothy, foul smelling stool) is caused by a decrease in pancreatic enzyme secretion and indicates impaired digestion and possibly an increase in the severity of the pancreatitis. the client should see the hcp

the client in end stage liver failure has vitamin K deficiency. which interventions should the nurse implement? select all that apply 1. avoid rectal temperatures 2. use only a soft toothbrush 3. monitor the platelet count 4. use small gauge needles 5. assess for asterixis

avoid rectal temps, use only a soft toothbrush, monitor platelet count, use small gauge needles: vitamin k deficiency causes impaired coagulation, therefore, rectal thermometers should be avoided to prevent bleeding. Soft bristle toothbrushes will help prevent bleeding of the gums. platelet count, ptt/pt, and INR should be monitored to assess coagulation status. Injections should be avoided, if at all possible, bc the client is unable to clot, but if they are absolutely necessarily, the nurse should use small gauge needles

Teaching in relation to home management after a laparoscopic cholecystectomy should include a. keeping the bandages on the puncture sites for 48 hours. b. reporting any bile-colored drainage or pus from any incision. c. using over-the-counter antiemetics if nausea and vomiting occur. d. emptying and measuring the contents of the bile bag from the T tube every day.

b Rationale: The following discharge instructions are taught to the patient and caregiver after a laparoscopic cholecystectomy: First, remove the bandages on the puncture site the day after surgery and shower. Second, notify the surgeon if any of the following signs and symptoms occur: redness, swelling, bile-colored drainage or pus from any incision; and severe abdominal pain, nausea, vomiting, fever, or chills. Third, gradually resume normal activities. Fourth, return to work within 1 week of surgery. Fifth, resume a usual diet, but a low-fat diet is usually better tolerated for several weeks after surgery.

the nurse is caring for a pt who had an open cholecystectomy 24 hrs ago. which of the follwing actions should the nurse take to assist the pt to maintain an effective breathing pattern? select all that apply a. place in a supine position b. provide analgesics for pain c. encourage coughing and deep breathing d. monitor bowel sounds e. assist with splinting during coughing f. maintain bedrest for 48 hrs after

bce: pain relief allows comfortable breathing, preventing shallow respiration's and guarding, splinting makes coughing more comfortable, encouraging coughing and deep breathing to keep lung clear

which assessment question is priority for the nurse to ask the client dx with end stage liver failure secondary to alcoholic cirrhosis? 1. how many yrs have you been drinking alcohol 2. have you completed an advance directive 3. when did you have your last alcoholic drink 4. what foods did you eat at your last meal

when did you have your last alcoholic drink?: the nurse must know when the client had the last alcoholic drink to be able to determine when and if the client will experience delirium tremens, the physical withdrawal from alcohol

A patient with pancreatic cancer is admitted to the hospital for evaluation of possible treatment options. The patient asks the nurse to explain the Whipple procedure that the surgeon has described. The explanation includes the information that a Whipple procedure involves a. creating a bypass around the obstruction caused by the tumor by joining the gallbladder to the jejunum. b. resection of the entire pancreas and the distal portion of the stomach, with anastomosis of the common bile duct and the stomach into the duodenum. c. removal of part of the pancreas, part of the stomach, the duodenum, and the gallbladder, with joining of the pancreatic duct, the common bile duct, and the stomach into the jejunum. d. radical removal of the pancreas, the duodenum, and the spleen, and attachment of the stomach to the jejunum, which requires oral supplementation of pancreatic digestive enzymes and insulin replacement therapy.

c Rationale: The classic operation for pancreatic cancer is a radical pancreaticoduodenectomy, or Whipple procedure. This entails resection of the proximal pancreas (i.e., proximal pancreatectomy), the adjoining duodenum (i.e., duodenectomy), the distal portion of the stomach (i.e., partial gastrectomy), and the distal segment of the common bile duct. The pancreatic duct, common bile duct, and stomach are anastomosed to the jejunum.

the client dx with end stage liver failure. the client asks the nurse,, why is my dr decreasing the dose of my meds? which statement is the nurses best response? 1. you are worried bc your dr has decreased the dosage 2. you really should ask your dr. i am sure there is a good reason 3. you may have an overdose of the meds because your liver is damaged 4. the half life of the meds is altered bc the liver is damaged

you may have an overdose of the meds bc your liver is damaged: this is the main reason the hcp decreases the clients medication dose and is an explanation appropriate for the client

the nurse is collecting data for a pt who develops jaundice and dark, amber colored urine. the nurse recognizes that which of the following is most likely the cause? a. encephalopathy b. pancreatitis c. bile duct obstruction d. cholecystitis

c: bile duct obstruction can result in jaundice and dark aber colored urine due to bile blockage

the nurse is caring for a pt with cirrhosis. the nurse would cautiously use sedatives for the pt based on which of the following? a. the livers ability to synthesize protein is altered. b. sedatives may increase the risk of jaundice c. sedatives are potentially toxic to the cirrhosis pt d. sedatives promote the conversion of ammonia to ammonium ion

c: sedatives are potentially toxic to the cirrhosis pt. due to impaired hepatic metabolism of these meds

a client is recovering from a gastrectomy. the nurse will have to administer medication through a nasogastric tube. prior to giving the medications, the nurse will: 1. irrigate the tube with saline 2. reposition the tube and reapply the tube 3. check the tube for placement 4. insert the tube 1 inch further

check the tube for placement: most institutions require a radiograph to check for placement

which gastrointestinal assessment data should the nurse expect to find when assessing the client in end stage liver failure? 1. hypoalbuminemia and muscle wasting 2. oligomenorrhea and decreased body hair 3. clay colored stools and hemorrhoids 4. dyspnea and caput medusae

clay colored stools and hemorrhoids: clay colored stools and hemorrhoids are gastrointestinal effects of liver failure

The nursing management of the patient with cholecystitis associated with cholelithiasis is based on the knowledge that a. shock-wave therapy should be tried initially. b. once gallstones are removed, they tend not to recur. c. the disorder can be successfully treated with oral bile salts that dissolve gallstones. d. laparoscopic cholecystectomy is the treatment of choice in most patients who are symptomatic.

d Rationale: Laparoscopic cholecystectomy is the treatment of choice for symptomatic cholelithiasis.

a pt with acute pacreatitis is NPO and has been receiving on IV hydration. which laboratory result indicates the need to consult the dietitian for nutritional support? a. potassium 4.2 mEq/L b. sodium 130 mEq/L c. fasting glucose 82 mg/dL d. serum albumin 2.9g/dL

d: low serum albumin level indicates malnutrition (normal albumin levels are 3.5-5.

the nurse is caring for a pt with chronic pancreatitis, the nurse would expect an elevation in which of the following lab tests? a. serum bilirubin b. serum calcium c. serum albumin d. serum amylase

d: the serum amylase level is elevated in chronic pancreatitis

a 38 y/o woman was admitted with a medical dx of cholecystitis. the nurse would expect the hx of her present illness to include intolerance to which nutrient? 1.carbohydrates 2. fat 3. protein 4. vitamin c

fat: inflammation of the gallbladder, presence of gallstones, or both interfere with movement of the bile from the liver or gallbladder into the small intestine, where it promotes the digestion of fats

a 32 y/o client had a cholecystectomy 2 hrs ago. the client weighs 250 lbs and had general anesthesia. the client is c/o pain. priority nursing dx at this time: 1. alteration in comfort 2. high risk for airway impairment 3. alteration in fluid volume 4. knowledge deficit

high risk for airway impairment: the airway is of highest priority in postop clients after general anesthesia

which statement by the client dx with hepatitis warrants immediate intervention by the clinic nurse? 1. i will not drink any type of beer or mixed drink 2. i will get adequate rest so i dont get exhausted 3. i had a big hearty breakfast this morning 4. i took some cough syrup for this nasty head cold

i took some cough syrup for this nasty head cold: the client needs to understand some types of cough syrup have alcohol and all alcohol must be avoided to prevent further injury to the liver, therefore, this statement requires intervention

the nurse is administering a pancreatic enzyme to the client dx with chronic pancreatitis. which statement best explains the rationale for administering this med? 1. it is an exogenous source of protease, amylase, and lipase 2. this enzyme increases the number of bowel movements 3. this medication breaks down in the stomach to help with digestion 4. pancreatic enzymes help break down fat in the small intestine

it is an exogenous source of protease, amylase, and lipase: pancreatic enzymes enhance the digestion of starches (carbohydrates) in the gastrointestinal tract by supplying an exogenous (outside) source of the pancreatic enzymes protease, amylase and lipase

a client has had gallbladder attacks in the past and is considered at risk for gallstone formation. the nurse should check that the clients diet is: 1. high in antacids and calcium 2. low in cholesterol 3. high in protein and carbohydrates 4. free of electrolytes

low in cholesterol: biliary calculi are high in cholesterol, therefore the cholesterol intake of the client should be limited

the client with an acute exacerbation of chronic pancreatitis has a nasogastric tube. which interventions should the nurse implement? select all that apply 1. monitor the clients bowel sounds 2. monitor the clients food intake 3. assess the clients intravenous site 4. provide oral and nasal care 5. monitor the clients blood glucose

monitor the clients bowel sounds, assess the clients intravenous site, provide oral and nasal care, provide oral and nasal care, monitor the clients blood glucose: the return of bowel sounds indicates the return of peristalsis, and the nasogastric suction is usually discontinued within 24 to 48 hrs thereafter. the nurse should assess for signs of infection or infiltration. fasting and the ng tube increase the clients risk for mucous membrane irritation and brk dwn. blood glucose levels are monitored because clients with chronic pancreatitis can develop diabetes mellitus

the nurse is discussing complications of chronic pancreatitis with a client dx with the disease. which complication should the nurse discuss with the client? 1. diabetes insipudus 2. crohns disease 3. narcotic addiction 4. peritonitis

narcotic addiction: its related to the frequent, severe pain episodes often occurring with chronic pancreatitis which require narcotics for relief

which task is most appropriate for the nurse to delegate to the unlicensed assistive personnel? 1. draw the serum liver function test 2. evaluate the clients intake and output 3. perform the bedside glucometer check 4. help the ward clerk transcribe orders

perform the bedside glucometer check: the UAP can perform a bedside glucometer check but the nurse must evaluate the result and determine any action needed

the client dx with acute pancreatitis is in pain. which position should the nurse assist the client to assume to help decrease the pain? 1. recommend lying in the prone position with legs extended 2. maintain a tripod position over the bedside table 3. place in side lying position with knees flexed 4. encourage a supine position with a pillow under the knees

place in side lying position with knees flexed: this fetal lposition decreases pain caused by the stretching of the peritoneum as a result of edema

The client dx with end stage liver failure is admitted with esophageal bleeding. the hcp inserts and inflates a triple lumen nasogastric tube (sengstaken-blakemore). which nursing intervention should the nurse implement for this treatment? 1. assess the gag relex every shift 2. stay with the client at all times 3. administer the laxative lactulose (chronulac) 4. monitor the clients ammonia level

stay with the client at all times: while the balloons are inflated, the client must not be left unattended in case they become dislodged and occlude the airway. this is a safety issue

During the evening following a partial gastrectomy, a clients oral temp is 100F. other data include a bp of 134/68, pulse of 88, and resp rate of 18. the nurse should: 1. notify the physician immediately 2. take temp every hour until it is normal 3. perform a respiratory assesment 4. remove the dressing and check the operative site

take temp every hour until its normal: the temp needs to be observed frequently to rule out some cause other than the inflammatory process

the client dx with liver failure is experiencing pruritus secondary to severe jaundice. which action by the unlicensed assistive personnel warrants intervention by the nurse? 1. the UAP is assisting the client to take a hot soapy shower 2. the UAP applies an emollient to the clients legs and back 3. the UAP puts mittens on both hands of the client 4. the cUAP pats the clients skin dry with a clean towel

the UAP is assisting the client to take a hot soapy shower: hot water increases pruritus and soap will cause dry skin, which increases pruritus, therefore, the nurse should discuss this with the UAP


Set pelajaran terkait

Opiates/ Drugs of Abuse/ management and titration

View Set

CBEST English Writing/ Reading For Ben

View Set

Chapter 10: Principles and Practices of Rehabilitation

View Set

ATTR/EXSC 25057 - Human Physiology - Exam 2 Study Questions & Answers

View Set

ITN 267 Legal Issues in Information Security

View Set

Chemistry, Chapter 2: Matter (BJU, Third Edition)

View Set

SDSU FA 17 - Homework_CH_02 - B A 370

View Set